You are on page 1of 111

Dr.

Gopika Sreedhar 18-06-2018

Hai all, these are the latest question papers which I referred. Please
confirm the answers given along with the questions before
referring, I didn’t make any corrections instead just copied the
papers and some don’t have answers.

This is a small contribution to all my fellow doctors within my little


knowledge.

Once again, a big thank you to all those who supported me and
made my exam a success.

Best of luck to all those who are heading for exams.

GOD BLESS….
Dr. Gopika Sreedhar
(my MOH exam date: 10/06/2018)

- - Questions almost what I remember - -

1. Patient upper anterior x-ray showing a notch in inter dental born between two central
incisors, it's due to

A. Thick frenum

B. Eruption cyst

C. Supernumerory tooth

2. Indirect retainer mostly needed in

A. Class4

B. Class 1

C. Class 3

D. Class 3 with modification 1

3. Child came to the clinic with his nanny and u notice very large scar in his forehead, u
will

a-Take history f scar from nanny and work

b-Take history from child and work

c-Call parent to take history and work

d-No work without parent

4. Ph of caoh

A- 6

B-8

C-12.5

D-9

5. Principle of GTR

A. Space creation and wound stabilisation


B. Surface biomodification and wound stabilisation

C. Bone remodeling and intranarrow penetration

6. Mature tooth with deep caries with pain

A. RCT

B. Pulpotomy

C. Pulp capping

7. Inflamed pulp and we remove 3mm pulp pathology

A. Partial Pulpotomy

B. Pulpotomy

C. Direct pulp capping

D. Indirect pulp capping

8. Inflamed pulp with periapical lesion

A. Pulpotomy

B. Direct pulp capping

C. Indirect pulp capping

D. Pulpectomy with znoe

9. Proxy brush used with embrassure

A. Type 1

B. Type 2

C. Type 3

D. Type 4

10. Most common types of occlusion which is easy to fabricate

A. Mutually protected

B. Unilateral

C. Bilateral balanced occlusion

11. Patient with 5 years old denture has a severe gag reflex, upon history he says he had
the same symptoms in the first few days of the denture delievery and it went all alone:

a. patient has severe gag reflex.

b. Patient has underlying systemic condition.


c. Denture is overextended

12. Simpifil type for canal enlargement by NiTi in:

a.Universal protaper

B. Reciprocal

c. Revers S

D. Light speed rotary

13. What is the copper ratio that eliminates gamma phase 2:

a 2%cooper

b 4%copper

c 10% copper

d 13% copper

14. Connects haversian system to osteocyte

A. Canaliculi

B. Volkmans canal

C. Interstitial lamelli

15. Which solution can corrosion tools

A. Sodium hypochlorite

B. Iodophor

16. Gutta percha disinfectant

A. H2O2

B. 5.2% NAOCL

17. In a clinical research trial we primarily need to :

A. Get written subject in the patient's own native language.

B. verbally write subject briefly

C. tell patients that they are part of study.

18. 51 year female came to u with complete denture 2 years ago. now suffering from
burning sensation u check the denture there was perfect seal and occlusal rest in a
position ......

the burning sensation from


A. viral infection

B. menopause

C. vitamin deficiency

19. Contraindication of epinephrine to pt. have?

A. Diabetic

B. Hypoparathyroidism

C. Hyperparathyrodism

D. Hyperthyrodism

20. 10 years child with congenital heart disease came for extraction of his lower 1st molar,
the antibiotic of choice for prevention of infective endocarditis is:

A. Ampicillin 30 mg /kg orally 1hour before procedure.

B. cephalexin 50mg/kg orally 1hour before procedure

C. clindamycin 20mg/kg orally 1hour before procedure.

D. Amoxicillin 50mg/kg orally 1hour before procedure.

21. Patient with perio problem lost some of his tooth and have chances of loosing again,
which type of denture to be used

A. Interim denture

B. Transitional denture

C. Immediate denture

22. A case with picture renal transplantation and patient has sore throat and burning?

A. Leukoplakia

B. Pseudomembranous candidiasis

C. Erythematous candidiasis

23. Characteristics of NUG

A. Non contagious

B. Pseudomembranous
24.

A. Residual cyst

B. Lateral periodontal cyst

C. Dentigrous cyst

25.

A. Picture of hemangioma

26. Picture of submandibular salivary gland

27. Picture of root caries

28. Minimum Crown root ratio

A. 1:1

B. 1:2

C. 2:3

29. Function of propofol

A. Increase intraocular pressure

B. Decrease intracranial pressure

C. Hyperventilation
30. Common Intracanal medicament

A. Caoh

31. What ib the property of nickel titanium wirre

A. Rigidity

B. Low coefficient of friction

C. Shape memory

32. 25 Yr patient deep pit and fissures, mild to moderate caries risk. Which of the
following not used for treatment

A. Sealant

B. Composite

C. Amalgam

33. Shoeing in complex amalgam restoration

A 1-1.5

B 2-3

C 0.5-1

D 1.5-2

34. Fluoride concentration in water

A. 1ppm

B. 2ppm

C. 3ppm

D. None

35. To avoid fracture of teeth after bicuspidization procedure the treatment should be??

a. splinting

b. bone with resin

c. extract and implant

D. crown

36. When you try to seat a crown on tooth you find open gap at the margin, you will

A. Remake a new Crown

B. Reduce inner surface of a crown


C. Fill the gap with composite

37. Under GA treatment pedo patient having mesial and distal carries on 2 adjacent teeth

A. Stainless steel Crown

38. Good implant material

A. Titanium

B. Zirconium

C. Co-cr

39. Which techniques will you use to anesthestize soft and hard tissues of mandibular
molars in one injection

A. Akinosi method

B. IANB

C. Gowgates

40. During clinical examination prior to compelete denture constriction the dentist ask pt
to say Ahh he noticed that the left half of soft palate is not moving the indicate the injury
of cranial nerve

A. Vagus nerve

B. Facial nerve

C. Glossopharyngeal

41. Orthognathic surgery,you plan to use 2 mm screw,the drill size to make a hole is...?

A 1 mm

B 1.5 mm

C 2mm

D 2.5 mm

42. Ph of enamel

A 5.5

B 6.5

C 4.5

D 7.5

43. Instrument which use for grasping a tissue when remove thick epulis fissuratum:
a- Allis forceps.

b- Addison forcep.

c- Curved hemostat.

d- Stilli forceps

44. G V black classification what number represent blade angle

A. no 1

B. no 2

C. no 3

D. no 4

45. Class 2, long face indicated for extraction premolar?

A. when flaring central incisor

B. not indicated

C. for deep bite

46. An old patient had a complete denture, he came to you after delivery complaining that
the lower denture moves from the ridge when he just moves his tongue, you put a (pip)
paste and checked the denture and there is no any areas of pressure or over extension no
occlusal discrepancy, what is the problem?

A. Under extension of borders

B. Over extension of borders

C. High occlusal plane

D. Cramped tongue

47. Question about vitality test

48. The following Structures opening into Middle meatus:

A. nasolacrimal duct

B. posterior ethmoidal sinus

C. maxillary sinus

D. sphenoid sinus

49. Although it is a caries detecting test, synder test is used primarily to

A.amount of saliva
B.number of acid producing microorganisms

C.ph in saliva

D.undetecting caries

50. Tooth present in 10yr old child

51. Complete denture patient tissue over mandibular Ridge is easily retractable, how to
manage

A. Minus surgical procedures to correct

B. Impression will replace tissue

52. Subgingival scaler

A. Gracey curette

B. Universal curette

53. canal filling using betta phase condensation gp:

A. Thermafill

B. Mc spadden

C. Obtra II

D. simplifill

54. Anomalis during initiation and profilration of tooth germ will lead to:

a. Amelogenasis imberfecta.

b. Dentinogenasis imberfecta.

c. Dentinal dysplasia.

d. Oligodontia.

55. 7yr old child Mesial and distal root fracture while extracting lower primary second
molar,how to manage?

A. Visualise and leave

B. Visualise and carefully remove

C. Remove after 1 week

56. Substantivity is the property to release when required from the oral structures,is the
property of which material

A. flouride
B. chlorhexidine gluconate

57. Main action of aspirin is:-

A. platelet aggregation

B. prothrombin

C. fibrenogin

D. thromboplastin

58. Not commonly used in endo dx:

A-ice

B-electric pulp test

C-hot water

D-percussion

59. Patency file is:

A. small file used beyond the apex

B. small rotary file

C. small file used with irrigant for recapitulation

60. Before placement of fissure sealant what should you do?

A. clean the surface with polishing paste

B. clean with polishing burs

C. occlusal réduction

61. Gingivectomy indication for all except :

A. Extensive alveolar removal

B. Eliminate supra bony pocket

C. Eliminate periodontal abscess

62. Born exposure diagnosed by ( not remembering full options, but answered this)

A. Cortical root exposure

63. Fracture in which one cortical plate is broken (something like that):

A. Greenstick fracture

B. Unilateral condyle fracture


C. Bilateral condyle fracture

D. Body fracture

64. Post graduate student use MTA the prognosis depends on prevent

A. Immediate future

B. Disturbance during closure of wound

65. Degree of saturation

A. Chroma

B. Value

C. Hue

66. Missing lateral incisor adjacent tooth is healthy and non restored

A. Implant

B. FPD

C. RPD

67. According to new technology finishing and polishing after amalgam filling

A. 24 hrs

B. 6 hrs

C. 2 days

D. 2 hrs

68. Young patient comes with subcondylar fracture, during examination you notice class l
occlusion. X-Ray showed gross condylar displacement. What's the management :

A. Open reduction followed by physiotherapy

B. Closed reduction followed by physiotherapy

C. nothing to do

69. Full length root viewed

A. Parallel technique

B. Bisecting technique

70. Xray needed to find correct position of foreign bodies or impacted tooth in Mouth

A. Tube shift techniques

B. Bisecting angle
C. Paralleling techniques

D. Two films against each other (something like that)

71. You want to make impression to patient and try to control fluid but bleeding doesn't
stop and provisional restoration to be made, what to do

A. Epinephrine retraction code

B. Wait for some days

C. Take impression with sodium chloride and polyether

72. Question about remineralisation

73. Blood glucose levels in patients detected by finger prick method

A. 100-200

B. 110-150

C. 40-100

D. 400-500

74. Name of dental carry system

A. DMF

B. MDF

C. MTD

75. best describe for prepared canal:

A. apical part is the narrowest in cross section

B. parallel wall terminates at apical collar

C. parallel wall few millimeters apical colla

EXAM -2
1. gates glidden drill
a. for coronal preparation of the canal
b. numbered from 9 to 20
c. for smoothening of line angles
d. gets entangled with dentin in root canal
2. radiograph of dentigerous cyst.. impacted 3rd molar..coronally attached radiolucency
3. radiograph of ameloblastoma...multilocular at the angle of mandible
4. pt came to a dentist with a tooth which got trauma 2yrs back,8yrs patient central
incisor, xray and radiographs made the dentist take up pulp revascularization
what is the present condition of pulp
a.open apex vital
b.closed apex necrosed
c.primary tooth necrosed
d.open apex non vital
5. right time for 1st orthodontic screening
a. 6yrs
b. 7yrs
c. 8to 9 yrs
d. 3yrs
6. most crucial phase for stoppage of thumb sucking habit
a. decidious dentition
b. early mixed
c. late mixed
d. pernanent dentition
7. patient with mesial distal and buccal caries on decidio central incisor. dentist decided
metal crown placement. what is the next step in management
a. selection of metal crown before caries removal
b. selection of metal crown after caries removal
c. does not matter before or after removal of caries
d. none of the above
8. the fluoride supplement most importantly depends upon
a. temperature of location
b. age
c. fluoridated water intake
d. form of supplement
9. dentist wants to prevent displacement of restoration of a short walled prepared cavity.
what is the most desired step that would lead to this
a. placement of retentive pins
b. placement of retentive grooves
c. retentive undercuts of internal line angles
d. . flaring of cavosurface margin
10. ph of caoh cement
a. 12
b. 12.5
c. 7
d. 10.5

11. what is the management of the tooth with fraCture of crown upto the margin of
alveolar crest
a. extraction
b. crown lengthening followed by post and core
c. subgingival tooth preparation to create ferrule effect followed by post and core
d. orthodontic extrusion followed by post and core12. forcep used for disimpaction of
fractured nasal bone
a . rows disimpaction forecep
b. william hayton forcep
c. walsham forecep
d. adsons forcep
13. which canal have the rarest probability of finding a second canal
a. mesiobuccal root of maxillary second molar
b. distal root of mandibular first molar
c. distobuccal root of.max first molar
d. mandibular central incisor
14. type of cementum in the coronal 2/3rd of the root
a. cellular extrinsic
b. cellular intrinsic
c. acellular extrinsic
d. acellular intrinsic
15. intraoral picture of hiv patient came to clinic for follow up with complain of burning
sensation in the mouth
showing white coloured lesions on the uvula and soft palate..
a. acute herpetic lesions
b. acute pseomembranous candidiasis
c. candida infected leukoplakia
d. lichen planus
16. max 3rd molar tooth goes into the upper space while extraction.what should be done
a. observe and follow up
b. ct followed by removal under g.a.
c. no treatment just inform the patient
d. explore the same site after 3weeks and remove the tooth
17. blacks instrument formula 8 - 40- 16- 14
what is the length of the blade
a. 8
b.40
c.16
d. 14
18. picture of upper arch..arrows on space between later incisor and canine.identify
a. leeway space
b. mixed dentition space
c. space of louis
d. primate space
19. a 35yrs old patient came for a routine checkup. on radiography a small radiolucency
with scalloped margins was seen periapical to the the tooth. no symptoms ,no pain on
percussion, associated tooth vital.
a. stafnes bone cyst
b.static bone cyst
c.unicameral bone cyst
d. simple bone cyst
20. a radioopaque radiolucent lesion was found during radiographic examination of the
patient. clinic examination reveal no expansion of the cortical plates no pain. the aspirate
was a straw coloured yellow fluid. it was confirmed to be ossifying fibroma.what is the
treatment
a. no treatment
b. wait till the growth ceases followed by smoothing of expansion
c. enucleation
d. marsupialization
21. an amalgam tooth restoration got fractured at isthmus . what is the most probable
cause
a. improper trituration
b. high occlusion
c. flaring of the cavosurface margin
d. over finishing
22. a young patient came with number of restoration and large number of new caries in
the mouth. on examination the gingiva appeared red and much swollen with a probing
depth of 2 to4 mm. what is the peridontal diagnosis of the patient?
a. high caries index
b. gingivitis
c. juvenile peridontitis
d. chronic periodontitis
23. what is the most coomon risk factor of fracture of anterior tooth
a. amelogenesis imperfecta
b. fluorosis
c. proclination
d. dentinogenesis imperfecta
24. why is premolar mostly at high risk of perforation during the endodontic treatment
a. it have more coronally placed furcation
b. presence of concavity on the immediate coronal part of root surface
c. wider canals with less surrounding dentin
d. less mesiodistal diameter
25. fracture case
pt recieves blow on the face causing orbital rim movement
open bite, movement at fronto zygomatic suture, orbital hyperteleorism, ecchymosis.
type of fracture?
a. le fort 1
b. le fort
c. lefort 3
d. isolated zygomatic fracture

28. patient came up with multiple caries in his mouth..on examination 5 endodontically
treated tooth and multiple areas of restoration were seen.what is the caries index of the
patient
a. moderate caries risk
b. high caries risk
c. mild caries risk
d. periodontal evaluation is necessary
29. what is the maximum safe dose of local anaethetic for a 65 yrs old patient of 80 kgs.
with 2% of lignocaine with 1:100000 epinephrine
a. 478
b. i dont remember options ...some in 500 some in 600 n 300s
30. one simple question on tongue tie
when is it treated
a. limited tongue movement
b. tongue touching palate
c. hyper mobility of tongue muscles
d. depressed tongue
31. patient came with rpd providing replacement to missing lower molars. on examination
it was found that it is an incorrect rpd design with no rest or support. what is this type of
rpd
flangeless
open faced
lingual
buccal
32. bur used for making v shaped rest on the cingulum of canine
a. round bur
b. tapered
c. fissure
d. inverted cone
33. which cement cause maximum pulpal tissue irritation
a. calcium hydroxide
b. zinc oxide eugenol cement
c. zinc phosphate cement
d. zinc polycarboxylate c
34. child came with tetracycline stains on his central incisors and molars
what is the probable age at which tetracycline was prescribed to this patient
a. 1yr
b.. 2y
c. 6yrs
d. 5 yrs
35. niti wire used in ortho .. reason?
something like that =》 i chose shape memory
36. most common malignant tumour of minor salivary gland= acinic cell
37. carbohydrate leading to dental caries depends on
a. frequency of intake
b. amount of "
c. form of "
d. duration
38. compound added in alginate to initiate the reaction
a. soduim phosphate
b. pottasium phosphate
c. sodium sulphate
d. calcuim sulphate

39. presence of microbes beneath the pit and fissure sealant will..
a. arrest the growth of bacteria
b. protect the growth of bacteria
c. induce growth of bacteria
d. reduce the growth of surface bacteria
40.cell rich layer
a.contain mast cell and fibroblasts
b. c fibres
c. mitotic activity...i chose this
41. pt came with a complain of heaviness related to maxillary premolar area. radiography
revealed vital tooth. both the premolars appear normal but show positive percussion test.
your diagnosis
a. chronic general periodontitis
b. lodgement of extraoral substance in gingival pocket
c. maxillary sinusitis
d. dental space infection
42.after providing adequate inferior alveolar block for extraction of mandibular 1st molar.
patient still complains of sensation.which nerve will you consider for the next block
lingual nerve
mylohyoid nerve
mental nerve
posterior alveolar nerve
43.odontogenic lesion which does not show any effect in radiograph is
a. acute apical periodontitis
b. odontogenic myxema
c. chronic periodontitis
d. alveolar osteitis
44.bone of 1st arch is
a. hyoid
b.maxilla
c. mandible
d. mental tubercle
45. growth of mandible begins at
a.4 to 8 weeks intrauterine
b. 8 to10wks iu
c. 10 to 12 weeks
d. in third trimester
46. crystals responsible for formation of enamel
a. hydroxyapetite
b fluoropatite
c. calcium oxide
47.needle gauge for fnac19gauge

48. when does there exist a gap between dentist and treatment
a.resources need and treatment
b. wide distribution of resources
c. needs meet demands
d...
49. after bicuspidsation what is the most step to maintain the integrity of the tooth parts
a. full crowns
b. short span bridge
c. long span bridge
d. none of the above
50. patient came with yellowish coloured tooth. radiographically reduced surface enamel
thickness with large pulp chamber is seen. your diagnosis
a. amelogenesis imperfecta
b.dentinogenesis imperfecta
c. regional odontodysplasia
d. dental fluorosis
51. one of the criteria for impression material is its hydrophilic ability to replicate the
tissue details even in presence of some moisture. which of the following impression
material is highly hydrophilic
a. polysulphide
b. polyether
c. addition silicon
d. condensation silicon
52. question about bull eye lesion
-> erythema multiformi
53. which of the following files have positive rake angle
a. k file
b. k reamer
c. protaper
d. pro files
54. which of the following is used for subgingival scaling
a. hoe
b. sickle
c. gracey
d. interproximal
55. conc of chlorohexidine in mouthwash
a. 1.2
b. 0.12
c. 12
d. 0.22
56. increased stress on periodontally compromised tooth is called
a. primary trauma from occlusion
b. secondary " " "
57. adults are less prone to caries progression as compared to children. this is due to the
fact..
a. adults have hard tooth
b. age related deposition of sclerotic dentin
c. children Eat sugars
d. adults are more prone to caries progression
58. MTA is most commonly used root canal sealant. mta is
a. mineral tetra oxi aggregate
b. mineral tri oxiaggregate
c. magnesuim trioxi amide
d. metal tetracycline and acid
59. veneer is contraindicated in
a. fluorosis
b. short tooth
c. bruxism
d. mouth breathers

60. fluoride supplementation in children with given level of water fluoridation depends
upon
a. temperature
b. type of supplement
c. age of patient
d. gender
61. question about indirect retainer
62. on denture delivery day the denture wearer patient have a space between upper and
lower denture equal to insertion of a wax knife when the other side is occluded.
the vertical relation and centric relation was devoid of any discrepancy during the try in
procedure. what is the next step
a. rebase
b. reline the denture
c. chair side selective grinding
d. articulating the denture again to check occlusion
63. long case on geographic tongue
64. ceramometallic crown preparation margin
65. maryland bridges are
a. conventional fpd
b. short span fpd
c. resin bonded fpd
d. long span fpd
66. minor connector are connected to major connector
a. at 90 degree angle
b. acute angle
c. obtuse angle
d. 180 degree joint
67. when porcelain tooth are placed opposite to natural tooth it causes
a. wear of porcelain tooth
b. wear of natural tooth
c. no wear of either tooth
d. fracture of the whole crown
68. system B uses
a. warm gp condensation
b. lateral gp condensation
c. cold condensation
d. single gp cone technique
69. a file have a diameter of 0.2 at the tip
how much should it be cut to get the diameter at tip of file number 35
70. best root canal sealer is
a. sodium hypochlorite
b. calcuim hydroxide
c. edta
d. rc prep
71. which of the following are effective against hepatitis b infection
1. iodophor and sodium hypochloride
2.. formaldehyde
3. ethylene oxide
4. gluteraldehyde
a. 1 ,2 and 3
b. 2 3 4
c. 1 and 2 only
d. 3only
72. radiograph of compound odontoma
73. increase in copper in amalgam alloy causes
a. delayed expansion
b. elimination of gamma 2 phase
c. increase in tarnish n corrosion
d. all of the above
74. periodontal attachment loss is measured from
a. gingival margin to pocket depth
b.cej to pocket depth
c. cej to the gingival sulcus
d. gingival margin to alveolar bone
75. mandibular 1st premolar is most likely to get pulp exposure during cavity preparation
because
a. small tooth
b. large pulp chamber
c. typical morphology
d. both a n b
76. type of bone best suited for implant placement
a. type 1
b. type 2
c. type 3
d. type 4
77. main cell responsible for periodontal modelling
a. osteoclast
b. osteoblast
c. fibroblast
d. odontoblast
78. forecep used for removing bony prominence during osteoplasty procedure
a. bone rongeurs
b. bone file
c. periosteal elevators
d. osteotomy forceps
79. the property of antagonising the clasp to prevent dislodgement in occusal direction is
called
a. resistance
b. retention
c. reciprocation
d. sealing
80. which property of wrought wire is better than its counterparts for construction of
clasp
a. retention
b. less stress on abutment tooth
c. shape memory
d . flexibility
81. primary support area for upper complete denture is
a. rugae
b. alveolar ridge
c. hard palate
d. buccal flanges
82. Hutchinsons triad is specific to
a. tuberculosis
b. primary syphillis
c. congenital syphillis
d. gardners syndrome
83-Clamp for partially erupted molar
14a
151s
53r
84-Grade II furcation involvement treatment of choice:
A- GTR (guided tissue regeneration)
B- full flap with curettage
C-periodontal therapy
85-Cutting edge of an ideal instrument should be
a. Parallel to long axis
b. Perpendicular to long axis
c. Perpendicular to shank
d. 3 mm from long axis
86-PDI classification for missing teeth in upper and lower arch including canines:
a)class 1
b)class 2
c)class 3
d)class 4
87- Rideal walker test
a. Used for selection of stainless steel crownsa
B. Used for selection of restorations
C. Used for selection of shade
D. Used for disinfection
88-What is the blade length of cutting instrument with the following formula 10-85-8-14 :
A-10
B-85
C-8
D-14
89-To remain stable , a rubber dam clamp must contact the anchor tooth gingival to the
height of contour. Which other criterion must the clamp satisfy?
a) All four points must be sharp
b) All four points must contact the tooth
c) The bow must be directed to the distal side of the tooth
90-Severly stained maxillary central with smal distal caries in 20 yr old what is the best
managment:
A- pfm crown
B-all ceramic crown
C-veneer ceramic
91-Which of the following conditions is highly indicated for the short therapy of DOTS and
is directly observed once in the clinic:
A) Tuberculosis.
b) AIDS
C.) Syphillis
92-Fluoride in dental office do not cause fluorosis because :
A Very little amount of fluoride
B Tooth already calcified
C Saliva wash the fluoride
93-process of removal of one root of a tooth to save the other by leaving the crown intact
a. root resection
b. root amputation
c. bicuspidization

EXAM -3
1. Parapost technique
2. Acromegaly
3. Shoeing of amalgam
4. Positive rake angle : a) k file b) k reamer
5. Fibrous dysplasia
6. Brown line on premolar, best way to reach diagnosis: a) leave and return after 6
months for extension of lesion****
7. Shape of max central and lateral access cavity
8. Epidemiology
9. Lefort 1, severe bleeding, which artery in injured?
10. Leukoplakia
11. Raspberry like lesion associated with denture
12. IANB needle length?
13. Pit and fissure probe? : a)blunt b)non forceful blunt probing****
14. Renal patient , dental clearance on : a) non dialysis day***
15. Provisional crown, occlusal high. We manage n reduce occlusal but it became
thin, why: A)dueto over eruption of opposing tooth****
16. Denture, whemever tongue elevate, denture moves up: a)under extension b)
cramped tonguec)over extension d)improper occlusal preparation

EXAM -4
1- A class II RPD diagram, showing an error in the diagram itself. And the premolars
were mobile. The error in?

1- Major connector*** They were too sub-gingival away from the back of teeth,
becuase it needs ligual plate.
2- indirect retainer
3- stress breaking release kind of that
4- the metal mush
The kind of bacteria in the saliva?
1-Actinomyces**** not sure
2-Kind of Vironella!!!! can't recall the name!!!!
3-Staph
4-Strept
After doing an electro-surgry on the gingiva, you have a width of gingival pocket
depth of ???
1- 3 all around the surface.*****
2- 3 all around the proximal.

Do your research. 3- 2......


4- 2......

The distance between 2 implants....3mm


The maxillary division enter Foramen
1- Ovale
2-rotundum
3-spinosum
4-........
The amount of amalgam ingestion per day? 3, the same as in the files
dentifrices-1000
Flouride in ppm-1
A long case, sever pain on the cheek on touching or eating
...
1- Trigeminal Neuralgia*****
2-Ramsay Haunt syndrome ( do your research)
3-
4-
A question about the ****extended flanges after 24 hours from wearig the denture
back to the dr. The same in the files
An ulcer on the lower labial mucosa....
1-Over-contoured flanges********
2-Over-extended flanges
3-......
4-........
What do we call an incidence of population divided on the diseased population?
wasn't mathematical at all
1-Prevail*******
2-...
3-....
4-...
The outcome of a disease in a group of diseased people?
Etiological
Risk
......
Prognostic****
A diagram showing a porcelain upper teeth opposing lower natural teeth.....
*******Abrasion
EBA, the same as in the files....
Unfilled****
.......
Primer and bonding agent
...........
A question about the Halogen light, what to do???? can't recall, but i read it
somewhere in the files...
Fix the battery
don't do anything******
......
.......
A very tricky easy q about the FREEWAY space, been written in a way to think its the
leeway space, because they've wrote "in each arch"!!! but they've asked about the
FREEWAY space... Read carefully
Do your research. As in the files.
A question about the infiltration in the upper first molar
*****ٍSupra-periosteal
A VIII patient.......Hemophilia A
How to treat the hypersesitivity? Wasn't an actual treatment, but more to the releasing
of it......Can't recall the answer, but the same as in the files
1- Root planning
2- Open up the tubules to release the intra-pulpal pressure*****

EXAM -5

1- feature that describe the energy absorbed by a material before deforming. -


toughness ?
2-crown to root ratio - 2:3
3-linchen planes histpothalogical characteristic.
4-best radio for proximal caries - bitewing
5-device to limit child extremities - papoose board
6-most recurrent cyst
7-complete blood count test
8-bicondylar fracture in child without displacement or malocclusion
9-where does the submandibular gland open in the mandible
10-how to fix lack of proximal contact after cementing a glazed porcelain crown
- composite ??
11-milkiy appearance of porcelain - over firing
12-best pit and fissure treatment - sealant
13-child 4 years old with oral candidiasis treatment
14-epitietial lining of maxillary sinus
15-question about apexogenisis
16-question about IANB
17-how much anaesthesia in 1 carpule of 2% lid with epi1:100000
18-best and first endodontic test - cold ?
19-question about symptomatic apical periodontitis
20-question about Herpatic gingivostomatis
21-management of herpatic HSV
22-effect lidocaine toxicity on the heart - bradychardia
23-which bur to prepare the groove in a 3/4 crown
24-asymptomatic tooth with vital pulp, distal caries and some incical chipping
restoration type - ?
25-purple lesion on crown and root of max incisor - external resorption ?
26-what causes white flakes on incisors after removing oath brackets - enamel
hypoplasia - food debris ?
27-which tooth root mostly displaced in max sinus
28- best material for condylar graft
29- difference between cartilage and bone growth patterns
30-gracey and universal curets differences
31-area specific scaler - gracey
32-primary use of a dental wax
33-what to check last during try in: -aesthetics -occlusion contact ??
34-plaque consist of - bacteria
35- bacteria for initiation and progression of bacteria
36-question about a SCC on tongue poorly differentiated ... - bad prognosis +
high recurrence
37-electricl pulp test affect - nerve impulse ?
38-main pulp neural cells
39-anestheisa is dependent on - lipid solubility
40-functional appliance - bionator
41-what to study before placing a functional appliance - skeletal age ? - dental
development stage
42-patient with a newly erupted molar - pit and fissure sealant
43-patient had PSAB and still feels pain, need endo, what to anaesthetise next -
buccal infiltration ? - palatal
44-best impression material - PVS
45-most biologically acceptable base - CAOH
46-patient after ortho treatment had an external resorption- CAOH
47-question about Cohort study
48-disadvantage of Mcspedden technique
49-ehtical question about informed written consent before start of a medical
study
50-question about ectodermal dysplasia + skin pigmentation - Amelogensis
imperfecta
51-question about crown and root hemisection
52-what to do after bicuspidization - crowning ?
53-when to manage accidental root perforation
54-question about he focal infection theory
55-question about sclerotic dentin protecting the pulp
56-how long to keep topical anaesthesia -1min ? - 2min
57-question involving rpd + medial undercut which wire
58-patient with RPD after 5 years all is good but denture displaced away form
tissue when depressed distally, indirect retainer is good- management ? -
rebase ? - duplicate
59-when to perform surgery after dialysis
60-definetion of maintenance after treatment
61-when to give antibiotic
62-principles of GTR - space creation and won stabilization
63-when to stop thumb sucking habit - early mixed dentition
64-pacifier effect on dentition -posterior cross bite + anterior openbite
65-endo irrigation solution
66- misdiagnosed in bicuspid area - mental foramen
67-patient with difficulty in phonetics - lower anterior teeth set higher than lip
line? - using smaller size mold for maxillary
68- in developed countries caries risk is reduced due to - artificial water
fluoridation
69-question about adding butmaide.. to acrylic to make a stronger denture for
the patient - high impact denture ?
70-question about winged clamp and rubber dam placement- placed together
71-question about conscious sedation
72-best anticarogenic sugar substitute
73-ovate pontic used for - max anterior teeth
74-question about pericoronitis
75-quesiton about raising a flap
76-injury to which area will cause lip paralysis - mental foramen area
77-when to do biopsy
78-after placement of temporary crown we did occlusal check and found
thinning of the crown whats the reason - inadequate tooth removal.
79-when to do lingual frenum management - limit tongue movement
80- advantage of RPI clasp - esthetic
81-a week after replacing amalgam with high zinc patient comes back with pain
the reason - delayed expansion
82-pulp polyp is usually - hyperplastic
83-which systemic disease cause gagging in denture wearer - ??
84-missing 13 which abutments used - 11,12,14
85-caries detecting dye - propylene glycol
86-amount of amalgam daily in

EXAM – 6

1-During try in of PFM bridge restoration the patient said he feels as if a seed is stuck
between his teeth this indicates:
A- Tight proximal contacts**
B-over extended margins
C-under extended margins
2- A small part of a periodontal curette wad broken and displaced in gingival
sulcus. how to retrieve it?
a- Schwartz periotriever**
b- tweezer
3-Difference between Gracey currete and universal :
1-Gracey for specific tooth area , universal for all surfaces
2-Gracey has one side cutting , universal is both sides cutting
3-Gracey cutting end offset angle is 70 currete , universal is 85
4-gracey is semicircular cross section , universal is triangular
A-1 and 2**
B-2 and 3
C-2,3, and 4
D-1,2 and 3
4-water irrigation device:
a- dilute bacterial products**
b- remove plaque
c- prevent attatchment
5-Surfactant usage
Increase surface energy**
Decrease surface energy
6-Difference between standardized and conventional gutta percha??
7-Instrument which use for grasping a tissue when remove thick epulis fissuratum:
a- Allis forceps.**
b- Addison forcep.
c- Curved hemostat.
d- Stilli forceps
8- pt with badly decayed upper 6 with mild pain, in x-ray there are series of radio-
opaque lines
a. paget's disease
b. osteosarcoma
c. garre's osteomyelitis**
d. fibrous dysplasia
9-Taper of preparation
10-Hyperactive immun reaction to rubber dam
11-Clamp for partially erupted molar
14a**
151s
53r
12-Pt with attrition what is seen in xray :
A-Pulp obliteration
B-hypercementosis **
C-external resorption
13-Recommendation for use of fluoride toothpaste for child under 3years old:
Recommended**
Limited
Toxic
14-Patient with Burning mouth syndrome , female has hot flushes, they show you
some test results: Vitamen B12 is and Ca slightly low among others, cause of BMS is?
a. Pagets
b. menopause. **
15-Grade II furcation involvement treatment of choice:
A- GTR (guided tissue regeneration)**
B- full flap with curettage
C-periodontal therapy
16-Surgery and recountouring under immediate denture, type of suture:
1) horizontal mattress
2) vertical mattress
3) interrupted**
4) figure of 8
17-Cutting edge of an ideal instrument should be
a. Parallel to long axis
b. Perpendicular to long axis**
c. Perpendicular to shank
d. 3 mm from long axis
18-Porcelain appears chalky cloudy color, what happened?
a.) Over firing**
b.) Under firing
c.) Excessive moisture
19-PDI classification for missing teeth in upper and lower arch including canines:
a)class 1
b)class 2**
c)class 3
d)class 4
20-Time to establish gingivitis by days:
5-7
7-14
3-5
14-21**
21-The movement of water across a selectively permeable membrane which needs
energy to be accomplished is called:
a. Osmosis.
b. Active transport.**
c. Filtration.
d. Diffusion.
22-Pt feel sever pain in upper mouth pain is radiated to ear and eye after you check no
caries when you press on 1st molar pt feel pain ?
Neuralgia
Maxillary sinusitis**
23- levator superior palpebral which nerve
Facial**
Trochlear
Ophtalmic
oculomotor
24-Child come to clinic with total reverse of upper anterior teeth
sometimes child discease his chin & lateral cephalometric give this
results Sna 80
Snb 82 Anb -2 what is diagnosis?
A-Class I malocclusion with skeletal class III
B-class III malocclusion with skeletal class I
C-class III malocclusion with skeletal class II
D-class III malocclusion with skeletal class III**
25-Pt cleft palate we start expansion the max :
1- first three months of normal max growth
2-after eruption of primary teeth
3-before three months from bone graft **
26-Best way to disinfect impression
A.Autoclave
B.Uv chamber
C.Disinfectant
D.Chemical sterilizer**
27-Spedding principle:
a. Used for selection of stainless steel crowns**
B. Used for selection of restorations
C. Used for selection of shade
D. Used for selection of sealant
28-Badly decay primary first molar possibly involving four walls. Best restoration
A- stainless steel crown**
B- Composite
C- Amalgam
29-Increased depth & rate of respiration is called:
a. Cheyne stokes breathing
b. Hyperventilation**
c. Hypoventilation
30-Periapical x ray
A patient had an endo trt on 47, 6 months ago with a temporary filling now he have
pain on biting. On the radiograph there is RL refered to 47 and on examination we
found a narrow deep pocket on the buccal and lingual side only. The 46 have a good
amalgam restoration from 15 years ago. What is your diagnosis:
A- microleakage in 47
B- vertical root fracture **
C- i can’t remember
31-Panaromic showing super numerary teeth and many unerupted teeth. Diagnosis
Ectodermal dyplasia
32-Periapical xray to a missed maxillary central with a odontogenic tissue
ressembling to a compound odontoma i cant find a similar x ray
33-Main Usage of sodium hypochloride
In dental clinic
34-Patient with mild pain present with a 15 treated endo from 3 years on radio graph rl
related to 15 : 1 acute apical abcess 2 chronic apical abcess 3 acute apical
periodontitis4 chronic apical periodontitis
35-7 y.o with thumb sucking, how to start treatment:
a- conseling**
b- psychiatric
c- orthodontic
36-What is the blade length of cutting instrument with the following formula 10-85-8-
14 :
A-10
B-85
C-8**
D-14
37-To remain stable , a rubber dam clamp must contact the anchor tooth gingival to
the height of contour. Which other criterion must the clamp satisfy?
a) All four points must be sharp
b) All four points must contact the tooth**
c) The bow must be directed to the distal side of the tooth
38-What anesthesia has most vasoconstrictor action?
A. Cocaine**
B. Tetracaine
C. Procaine
D. Articaine
39-During taking biopsy from posterior third of tongue, how to hold the anterior part
of the tongue:
a- towel**
b- appliance
c- no need
40- atraumatic restorative treatment, is defined as “a dental caries treatment
procedure involving the removal of soft, demineralized tooth tissue using hand
instrument alone, followed by restoration of the tooth with an adhesive restorative
material:
Alternative restorative treatment
41-Cementosis and ankylosis of the teeth is common features of:
1. Cherubism
2. Osteomyelitis
3. Paget's disease **
42-Anterolateral area of hard palate submucosa contains:
• mucous glands
• serous glands
• adipose tissue**
43-Severly stained maxillary central with smal distal caries in 20 yr old what is the
best managment:
A- pfm crown
B-all ceramic crown
C-veneer ceramic**
44-Difference btw chronic and acute apical periodontitis :sinus tract
45-Type of bone best for implant ?
type 1
type 2**
type 3
46-LEAST Implant Success in bone:
a. type1
b. type2
c. type3
D. type 4 **
47-carbohydrates affect caries mostly by:
a. frequency**
b. quantity
c. consistency
48-Which of the following conditions is highly indicated for the short therapy of DOTS
and is directly observed once in the clinic:
A) Tuberculosis. **
b-HIV.
C) H1N1.
D) Mental illness
49-Lateral condylar guidance :
A. L=H+12/8
B. L=H/12+8
C. L=H+8/12
D. L=H/8+12**
50-pregnant women in second trimester came to your clinic , what is the best local
anesthesia ;
1- articain
2- lidocaine **
3- bupivacaine
51-Fluoride in dental office do not cause fluorosis because :
A Very little amount of fluoride
B Tooth already calcified**
C Saliva wash the fluoride
52-Attachement level is the
distance from:
a-CEJ to pocket depth
b-CEJ to mucogingival junction**
53-Associated with renal failure :
A- hyperthyroidism
B-hyperparathyroidism
C-secondary hyperthyroidism
D-secondary hyperparathyroidism**
54-process of removal of one root of a tooth to save the other by leaving the crown
intact
a. root resction
b. root amputation **
c. bicuspidization
55-surfaces visible in FOTI
A. All surfaces of all teeth**
b. all surfaces of anterior teet
c, proximal surface of anterior
d. proximal surface of posterior
56- the needle is parallel to occlusal plane during anesthesia:
A- gow gates technique
B- vazirani akinozi technique**
C- IANB

EXAM - 7

1. Bacteria in Osteomyelitis,
A. staphylococcus***
B. Streptococcus
2. Long qstn clinical presentation, with opg & occlusal view___ buccal bifurcation
cyst
3..qstn related to fracture of condyle
4.Regarding needle in vazironi Akinosi techniques
A. 25 guage short needle
B. 25 guage long needle***
5 .Residual cyst image_ [opg]
6.qstn related to osseo integration in implant
7.controll tooth no TOP
cold 5sec,
heat 3sec,
EPT 15sec
Test tooth severe pain on Top
Cold no response
Heat no response
Ept no response
Diagnosis a. Reversible pulpitis
B. Irreversible pulptis
C. Vital pulp
D. Necrosed ***
8.qstn related to fracture of ethmoid bone
9 .Management of coronal 3rd # in pedo
10. Most effective preventive messure for pit and fissure __sealants
11. Long qstn, (attrition , pulpal obliteration, sibling lso has same problem) __
A. dentinogenesis imperfecta
B. Amelogenesis imperfecta
12 . Patient with severe pain & fluctuate swelling,patient not allowed to touch the
tooth emergency management?
A. Incision and drainage
B. Complete debridement, incision & drainage
13 .systemic Antibiotc indicated in___
A. cellulitis **
B. Apical periodontist
14. Management of thumb sucking__early mixed dentition
15. Activation angle of blade__45 - 90
16. Maryland bridge__resin bonded
17.La used in pregnancy___lignocaine
18. Action of vasoconstrictor in la_
A. increase duration & increase intensity of LA**
B. Decrease intensity of LA
19. Resorption due to ortho management___caoh2 dressing
20 .patient with trauma of tooth, non lingering pain to cold, tenderness on percussion
diagnosis
A. Asymptomatic irreversible pulpitis, symptomatic apical periodontist
B. Asymptomatic reversible pulpitis symptomatic apical periodontist
C. Symptomatic reversible pulpitis, symptomatic apical periodontist
21 crown 10mm, root 15mm, asking about crown, root ratio___ 2;3
22. Qstn related to opg
23.Last step before cementation of crown____ polishing of metal
24. shape of acess openg of pm___oval
25 .long Clincal qstn regarding veneer
26.Gingivectomy__ to eliminate pseudo pocket
27.Dentinal changes in deep caries
28. 2nd best xray for implant
29. Qstn related to Step back technique
30. Dibetic and hypertensive patient has lesion
1 *2 cm on lateral border of tongue, type of biopsy
A. excision
B. Incision
31.missg 6, tilted 7, planning for fpd, best management
Ortho extrusion of 7
32.Material used in preventive resin__
33.Smoking & non smoking gingivts__less intense in smokers
34.steps of sterilization
35. qst related to Green stick fracture
36. Clean and dry cavty
37.Utility wax main ingredient
A. Paraffin***
B. Gumdammer
38.Force applied to pdl during orthodontic treatment___ light force
39. Curing light 450nm what is the decision nothing to do
40. Instrument used in gingival bevellg in class 2 ____GMT
41. Qstn related epoxy resin, type of impression material ____poly ether
42. Abcess incision
A. no 11 blade***
B. No 12
43.patient on warfarin about to do surgery for him, which of following is more
important
A. Pt***
B. Ptt
44. principle of GTR
A. Space creation & wound stabilization ***
B. Root biomodification & wound stabilization
45. intrusion type of resorption ____Inflammatory resorption
46.. long case history[ fever, malaise, gingivitis] ___Hsv
47.. Discoid cleoid instrument___ Amalgam carving42.long qstn about addison
disease
43.recent advantage of Diagnodent
44. Qst related to Fibro optic
A. Early detection of superficial caries
B. Deep caries
C. Tooth fracture
45.qstn related to flap
46. Most retentive crown__Full metl crow
47.most favourable taper of rpd __
A. 0.010**
B. 0.020
48. Ameloblastoma follow up__10years
49.property of gic__fluride release
50. Water fluoridation___1 ppm
51.Decreased alkaline phosphates, ealry loss of lower anterior
52. Long qstn related to Papillary hyperplasia

EXAM – 8

1...Quadlock devise
-Fixed dentoalveolar expansion @
-Fixed facial alveolar expansion
2...ANUG
-Pseudomembranous @
-Vesciles
3 ...gingivectomy
-Subrapeiodontal abcess
-Invasive alveolar surgery
4 when do first time use toothbrush
- when first tooth erupt
- when 2 years old
5 image with caries on molar with periodontal recessions and caries under CEJ what
type of caries
-occlussial
-proximal
- root caries @
6 fibrous dysplasia
- enucleation ??

EXAM – 9
1-Cementum in coronal 2/3 have:
d. Acellular intrinsic fiber
c . Cellular mixed fibers
b. Acellular extrinsic fiber **
d. Intermediate cementum
Proxy brush with which type of embrassure: type 2
3.Streptococcus mutans initiates caries but Lactobacilli progress caries to
cavitation

4.After appling porcelain over metal and firing cycles , porcelain found cloudy and
milky what is the cause
-over firing
-under firing ***
-over opacities
5. broken cusp upper premolar not carious not restored with cold water has brief
pain what’s the pulp status irreversibly inflamed reversibly infamed
innervated A delta fibers >> my answer ( no reason :D)
6.With two wall defect (osseous crater) better use freezed dried
bone decalcified cortical freezed bone
decalcified cancellous freezed bone
7- Multiple sebaceous glands cysts and supernumerary teeth and other
manifestations -gardner’s syndrome
8- To increase bone density in graft site
-bone crib
-Devascularized cortical and cancellious bone
-vascularized cortical and cancelous
9- Patient with epidermolysis bullosa what oral manifestation hypophosphatsia
amelogenesis imperfecta ??? it ‘s said hypoplastic teeth in mosby book so I chose
it dentinigenesis imperfecta Osteogenesis imperfecta
10- Dental bud at witch weak intra uterine 4 - 5 - 6 - 8
11- two central incisor in 4 years boy intruded 4-5 mm to follicle of permanent whats
the ttt
-leave and observe
-reposition and splint
-remove as quickly as possible **
12- disinfect gutta percha
-naocl **
-hot oven
-autoclave
13- child with multiple RL in lower mandible in ramus area expanding jaw and
making lower 7 follicle become advanced under lower 6 -cherubism -fibrous
dysplasisa
14- Facial nerve supply which
-buccinator **
-med pterygoid
-lateral pterygoid
15- cranial nerve sensory for orofacial area
-VII
-trigiminal V **
-IV
16- dental floss to -drisrubt plaque formed
17- brushing is to
dirupt plaque matrix formed **
-remove remaing food on teeth
18- mother calls u saying her child swallowed 50 mg fluoride paste what to do
-leave and observe manifestations -have acidic drinks and go to emergency -have
calcium and go to emergency **
19- muscles of long midface to short midface
-increase -stronger -weaker ??? my answer but not sure
-the same
20- to re endo and remove silver points by
using hemostat or pliers
21- in lower lateral incisor with 1 mm remaining tooth structure over gingiva what is
used -custom made post w core
-carbon fiber post with composite core
-threaded post with amalgam
22- facial nerve supplies
-buccinator **
-lat pterygoid
-med pterygoid
-masseter
23- functional appliance
bionator **
-bite block
two other options
24 – to preserve root from peroforation and filling material integrity what should be
used to remove gutta percha from root for post application
-drill -heat ** my choice
chemical solvent

25- posterior bite block for treatment of


-anterior cross bite with deep bite
26- child with pain in lower E and
dentist removed 1-5 mm of pulp
what is this called
-pulpetomy
-partial pulpetomy
-pulpectomy
-apexification

27- picture of upper jaw only and


arrows mesial to upper
primary canines
-primate space

28- Bacteria grown in plaque communicate with each other


through
-quorum sensing *** not signaling
-signaling
-maturation
29-difference between the alveolar epithelium and the gingiva epithelium
is a-Absence of stratum spinosum b-Absence of stratum
granulomatous C-Absence of stratum cornium **

31. case about female patient came after receiving denture complaining about
inflammation in her lower anterior area under denture with some ulceration
asking about what type of ridge she has
-flabby ridge
-khife edge ridge ** my answer
-flat ridge
32.most caries prone surface >> Buccal surface of maxillary posteriors
(maxillary first molars)

33-clamp used for partially erupted tooth?

a. #14 A. b. #W4A.

EXAM – 10

1.which compartment contains body fluids ..intestitial , intercellular ,


intravascular , transition compartment
2.adverse effect of cyclosporins ..staining , gingival hyperplasia , mucosal
discoloration
3.long span bridge ..high strength and high ridgity
4.limitation of using EPT..open apex, preganacy , narrow ya curve canals
_5.before application of pit and fissure apply..phosphoric acid , 3% hydrogen
peroxide
6.Before applying fissure sealents in primary cavity .apply varnish , calcium
hydroxide , clean n dry
_simplifil question..universal protaper
_ substansitivity..chlorhexidine gluconate
_medicine causing gingival enlargement ..phenytoin
_ which is not pulpal test ..percusion , EPT
_long question about missing 38 carious 36 same as file..mutlilicular
radiolucency..ameliblastoma
_question about lesion in junction between hard and soft palate..necrotising
sialometaplasia
_melonotic pigmentation kn skin and renal problem..addison disease
_copper lesion , hearing loss, notch incisor..congenital syhilitis
_question about simple bone cyst long case same as file.
_firm fixed nodes ..SCC
_Patient came hspitle fracture of mandibular symphysis wire used..6 inch 26
guage
_dentifrices ..1000ppm
_gates galliden bur used ..enlargement of coronal structure
_defination clinical attachment level..cej to pocket depth , cej to mucogingival
_how can we check periodontal disease progress...by attachment level
_differentiate between endodontic and nonendodontic periapical
lesion..radiogeaphic appearnce , EPT
_differentaite between vital and nonvital pulp ..EPT

EXAM – 11

1. case about class III malocclusion , what is the treatment ---> Advancment of
Maxillary Anterior (incisors)

2. class III restoration , which bevel ---> long irregular* , hallow smthng {i dont
remmber the options}

3. fluoride content in dentifrices --> 1000

4. case about pt given Diazepam IV feels burning sensation , its caused by --->
presence of propylene glycol { not sure check plz}

5. Pic of protruded upper and lower central incisors , after treatment its back to
position , what type of movement used -->

6. in a patient 9 yr when will the diastema closes ---> eruption od permenant


canine

7. Q about REATAINER in fpd and rpd

8. restorative material to be glazed to avoid dehydration ---> GIC

9. CASE (same like files ) ---> Trigeminal nuralgia

10. CASE pt have renal problem , and confusion ..... ---> hyperparathyroidism
11. raiograph ,Pt with blue sclera teeth , with multiple fracturs ---> DI , OI { i
choose DI but i think its OI since the patient have multiple fractures}

12. when scalining , the angle b/w scaler and facial surface of tooth ---> 45:90

13.case about patient surffering from severe pain , awaken him at 2 am cant
sleep ---> irreversible pulpitis

14. epithelization of gingiva ---> 7-14, 5-21

15. DI occurs in ---> histodifferentiation

16. amalgam in retrograde ---> zinc free , copper free , { i dont remember the rest
i chhose zinc free plz check}

17. radiograph , pt having RCT on 47 and amalgam on 46 , with pocket 6-7 mm


related to 47 ---> vertical root fracture

18. before filling in pedo ---> cavity varnish

19. attrition of upper and lower bicuspids , what will u see in the radiograph --->
hypercementosis { not sure }

20. full arch extraction , next step ---> interupted suture , ... { there was long
options i choose the one with intrupted sutures}

21. determining working length in endo rule ---> curved file to locate canal * ,
bisecting angle parallel

22. 20 file ---> 0.2 mm

23. pain definition ---> unpleasant sensation ....

24. etiological factor definition --->

25. cohort group definition --->

26. pic white discoloration due to ortho treatment , patient want esthatic, we will
use all except ---> coservative composite* { all othere options were veneer}

27. how to icrease ZO working time ---> a.add eugenol drop , b. mix on cold glass
slap , c. mix on paper pad { i choose eugenol drop not sure }

28. prognosis decrese ---> necrotic pulp with periapical lesion

29. proliferation of tissue under CD ---> surgical excision

30. amount of vasoconstrictor in LA ---> 36mg

31. 1st thing to do when removing Rubber dam ---> cut septal

32. tug back --->

33. indirect pulp capping used when ---> reversible pulpitis

34. most common ---> cleft lip and palate


35. case about missing 18,17,16 , and missing 24, 26 , there is carious 25 to be
extracted , which class is it after extraction of 25 --->

a.class I mod I

b.class I mod II

c.class II mod I ** { plz check }

d.class II mod II

36. imediate denture purpose

37. rest function

38. diagodont

39. most affected cell by radiosensitive ---> forgot the options :(

40. anterolateral in palate ---> taste buds ,

41. after GA ....... for restorative treatment, pt is preped and anasthesied ,what s
next step ---> take raiographs , give prophylactic AB

42. analgesic for child pt with renal disease ---> acemenophen (check the dose )

43. deflection of FPD (dont remember options)

44. pic of implant analogue

45. calcium hydroxide used b/w visits ---> antimicrobial affect

46. cleft palate occur in ---> 8-10 week IUL

47. gingivectomy indication all axcept ---> excessive osseous problem

48. amalgam cavosurface ---> 90'

49. open margin crown ---> remake

50. pulp polyp associted with ---> a.reversible pulpitis b.irreversible puplitis
c.necrotic pulp d.non-vital { i choose b but am not sure}

51. enamel can withstand forces ---> a.enamel tufts , b. enamel lamella , c. g..
enamel { check the answer }

52. autoclave ----> moist heat under pressure

53. cell wall biosynthesis ---> penecillin

54. composite matrix ----> BisGma

55. long Q about defeciency in factor VIII ----> Hemophelia A

56. substantivity .... (like files ) ---> chlorohexidine glugonate

57. in apicectomy the angle should be ---> acute


58. hypertensive pt 210/100 ----> reschedual the appointment

59. bacteria comunicate with each other ---> courum sensing

60. case about condyle displacment best ---> reverse town

61. how many time should complete denture be washed ---> a. after every meal
* b. twice c. thrice { plz check }

62. angular chelitis in pt wearing CD since 5 years ---> high intercuspal distance

63. thumb sucking causes ----> ant openbite post crossbite

64.single implant consideration ---> antirotation coronal somthing , { i dont know


the ansewr }

65. most common benign tumor of salivary gland ---> pleomorphic adenoid

66. case about CONDENSING OSTITIS

67.function of vasoconstrictor ---> reduce toxicity

68. pt cleft palate and lip , pit lip , anodontia ..----> van der waund

69. case about fracture in nose , which forcep ---> walsham's forcep

70. addind surfactant to irrigation ---> reduce surface tension

71.case of avulsed tooth , how long do u splint ---> a.1* b.2 c.3 d.4

72. ceramic primer ---> silane coupling agent

73 & 74. drug causes gingival enlargment ---> phenytoin {this q was asked twice}

75. subgingival inst ----> gracy

76.what make priority to private clinic than community ---> need assistant
outcome { in files they chhose assistant but i choose the outcome so check it }

77. with acute pericoronitis , managment ---> rinsing swap antibiotic

78.fluoride used in clinic ---> duraphat

79.mother call ....... asking about avulsed tooth ---> cool milk

80. anasthesia not working for lower molar periapical surgery ---> due to
decreased ph

81. prefered root for abutment ----> multiroot

82. sudden sharp pain which nerve ---> A

83. pt with paralysis of left side for 15 days includes eye , upper lower lip,
anterior third of tongue cannot raise eyebrows , which injury of facial nerve -->
a.injury to parotid gland

b.chondra tympani

c.upper part of facial nerve ** (not sure)

84.pt with deep pit and fissure 25 yr have high careis risk ---> sealant

85. best treatment to prevent caries ---> sealant

86. pt have white spot on buccal surface of molar , caries risk ---> high*,
moderate, mild

87. using fluoride in dentifrices, supplement dose not cause florosis ---> teeth
already calcified

EXAM – 12

1.simplifil apical seal used with which system:


A. Protaper
B. Reciprocal
C. Revo S
2.what kind of periodontal probe is used in the furcation area?
A. WHO.
B. Naber’s probe
C. UNC 15
D. Michigan
3.what is the radiographic tech being appropriate if used properly while planning for implant?
A. occlusal
B. periapical
C. Panoramic
D. CT
E. MRI
4.fiber optic diagnosis is
A. Quantitative
B. Qualitative
C. Qualitative and quantitative
5.Pt. presented to u having root recession he has pain when putting probe gently on the root what
is the diagnosis:
A. Dentin hypersensitivity.
B. Reversible pulpitis.
C. Irreversible pulpitis.
D. Apical Periodontitis
6.what is the mostly observed tissue response of successfully following oral hygiene instructions?
A. reduced pocket size
B. reduced plaque index
C. reduced bleeding
D. reduction in swelling
7.70 yrs. Old male pt. Comes to restore his badly decayed upper second molar .chance of involving
pulp by the infection from dentin compared to young patient?
A.Progress slowly in adult
B.Adult suffers less pain compaired to young
C.More progreesing in old patient than young
D.higher pulpal involvementhavhe less pulp affection
8.Patient with roughness on skin and shiny palms widening of pdl space but with no ridge
restoration and there is a bilateral destruction in angle of mandibular bone ,what is your
diagnosis:
A.Neoplasm
B.Scleroderma
C.Hyperparathyroidism
D.Aggressive periodontitis
9.diameter of number 20 gutta percha
A. 0.20mm
B. 2.0mm
C. 020mm
10. how long is the rinsing time of chlorhexidine in mouth wash to be effective
A. 15 s
B. 30 s
C. 40 s
D. 50 s
11.concentratoion of chlorohyxidin mouthwash is
A. 1.2%
B. .12%
C. 2.1%
12 .What is the important biomechanical propeRty required for single missing tooth implant
A. abutment made up on titanium alloy
B. abutment should be made in two parts
C. abutment should restrict the coronal rotation
13.Class 2 malocclusion with long faceorthodontics extract upper per molar
A. relieve incisor flaring
B. make incisor large
C. extraction isn't indicated
14.What different between center of the growth and site of
growth?
A. Independent
B. Centered
C. The center of growth is rapid
15.Radiographic criteria used for evaluating success endo therapy
A. reduction of size periapical lesion
B. no response to percission and palpation test
C. extension of sealer cement through laterla canal
D. non above
16.Avulsed tooth best medium
A.pt saliva
B.milk at room temprature
C.milk at cold temprature
D.saline
17.Oral manifestations of HIV in a child
A.parotitis
B.kaposis sarcoma
C.herpes gingivostomatitis
D.candidiasis
18.Spedding principle:
A. Used for selection of stainless steel crowns
B. Used for selection of restorations
C. Used for selection of shade
D. Used for selection of sealant
19.Mature tooth with caries expo of pulp treatment by
A. RCT
B. Pulpectomy
C. pulpotomy

20.identify the lesion(long question)

A. Radicular cyst
B. Nasopalatine duct cyst
C. Dentigerous cyst
21. Long question about submandibular sialolith and somewhat same pic

22 identify the local anesthetic technique

23.Impression material most stiff is:


A. Poly ether
B. Alginate
C. Poly vinyl siloxane
D. Agar-Agar
24.matrix and patrix part of (question was not direct like this)
A.Clasp
B.Attachment
C.Major connector
25.Disadvantage of full thickness mucoperiosteal flap
A. Delayed secondary healing
B. Scar tissue formation
C. interdental papilla integrity
26.Periodontal flap in which epithelial lining of periodontal pocket gets converted into attached
gingiva?
A. Modified Widman flap,
B. Apically positioned flap,
C. Undisplaced flap,
27.Functional appliance
A. Bionater
B. Posterior bite plane
C. Head gear
28.4yr old with 5mm intrusion of upper incisor which touching the permanent follicle what do you
do?
1. Wait and see if erupts
2. Extract carefully
3. Crown lengthening
4. Reposition manually and splint

29.the action of reciprocal arm


30.what is the most effective powerful hemostat with heavy bleeding after extraction
A. Cotton soaked with epinephrine
B.Oxidized cellulose
C.gelatin
D.hemcan
31.Order of placing of winged clamp type rubber dam
A.clamp before dam
B.clamp aftr dam
C.clamp and dam together
32.patient with multilocular radiolucency in mandible angle and multiple fractures
A.myloma
B.osteomyelitis
C.hyperparathyroidism
D.florid hypoplasia
33.According to CDC , to clean instrument before sterilization we use :
A. chemical
B. ultrasonic cleaners
C. blood dissolving solution
D. hard brush
34.avulsed tooth remain for 60 min what should do :
A. immerse in sadium hypo
B. in sadium flouride
C. in sadium flouride then hypochlorite
35.The main advantage of immersion technique is?
A. Prevent the impression from distortions
B. Helps all the surface of the impression to be disinfected
36primary teeth occlusion that develop CIII:-
A. Mesial step more than 2 mm
B. Distal step more than 2mm
C. End on
D. Edge on
37. patient came after avulsed tooth managed what's you appropriate time of splinting:
A. 1-2weeks.
B. 2-3 weeks
C. 3-4 weeks
38.A 60 year old woman comes to the clinic with complaint of angular cheilitis and she has been
wearing the same denture for last many years. What is the reason for this?
A. Decreased Vertical dimension
B. over extended denture flange
C. under extended denture flange
D. something about wrong eccentric position
39.A young 14 year old patient with excessive plaque and calculus and u decide to do scaling for
him.what is the best ultrasonic to be used which moves in an elliptical motion
A. piezoelectric
B. ultrasonic
C. magnetostrictive
D. sonic
40.Retention of veneer
A. preparation of tooth
B. Micro mechanical due to etching enamel & veneer
C. mechanical retention
D. groove
41 upper molar anesthesia in child
42.Tuberosity technique for block :
A. P.s.a nerve
B. m.s.a nerve
C. maxillary nerve
43.43 years old patient is scheduled for extraction of grossly decade maxillary premolar, clinical
history reveals that the patient has increase in atypical plasma esterase which of the following
local anesthesia agent maybe safely administered to the patient:
A. procaine HCL with 1/200,000
B. Propoxican HCL with 1/50,000
C. Prilocaine HCL with 1/280.000
D. Procaine HCL without adrenalin
44.Material for Chair Side relining of denture?
A. Soft liner
B. Light cure acrylic resin
C. Wax
D. Acrylic
45.Angulation of the face of blade of the scaler to stone during sharpening :
A. 50-60°
B. 70-80°
C. 100-110°
46.Decalcificiation in case of fixed orthodontic appliances occurs most commonly on which surface
of tooth?
A. Lingual
B. Inter proximal
C. Just behind the brackets
D. Around the brackets
47.Pt. Had undergone radiotherapy before 10 months. To make impression of removable partial
denture what is type of material
A. plaster of Paris
B. Rubber based
C. compoundd. ZOE
D. alginate
E. agar-agar
48.pt came with sever pain on his first permenant molar &with routine examination
dentist found white wrinkled ulcer on buccal mucosa ask pt‫ ا‬say that tobacco consumer since
10years what to do
A. refer to pathologist to take biopsy
B. follow up after 2weeks
C. topical anaeshesia &follow up
D. Its just a tobacco pouch no treatment needed
49.bad taste and smell in the patient's mouth, there are bubbles in the retiner cervical area,
diagnosis is:
A. loosening of the retainer
B. more occlusal forces
C. food imapction under the retainer
D. breakage between connector and retainer
50 .Pt underwent renal transplantation ions 3years ago he white non scrap able lesion on the
lateral side of tongue appeared corrugated and he has shaggy and frayed whats your dxs?
A. hyper plastic candidiasis
B. idiopathic leukoplakia
C. lichen planus
D. hairy leukoplakia
51.what does Enamel bonding agent (EBA) consist of:
A. Unfilled resin
B. Primer and bonding agent
C. A mixture of resins in an acetone or ethanol solvent
D. A wetting agent and resins
52.Indirect retainer placed?
A. Near the fulcrum line
B. Near gingiva.
C. Near edentulous area.
D. away from the fulcrum line
53.when patient says ‘ahh’ half of the soft palate is not moving.which nerve we test:
A. Glossopharyngeal
B. Vagus
C. Hypoglossal
54.Most destructive finish line:
A.shouldeR
B.chamfer
C.feather edg
55.Important sign of fracture in the body of mandible
A. Upward shift
B. Parasthesia of lower lip
C. Medial shift
56.The most destructive occlusal interference is
A. Centric
B. Working
C. Non working
D. protrusive
57.The usual cause of contacting or clicking posterior teeth :
A. Decreased vertical dimension
B. Increased vertical dimension
58.if the crown accessibilty is less during endo and limited removal of pulp and debris can cause
A. Crown fracture
B. Crown perforation
C. Discoloration
59.8 year Patient came to your clinic has impairedhearing, upon examination his mouth you found
copper color lesion , notchedincisor and mass on the occlusal surface of the molars . this patient
has :
A. Congenital syphilis
B. Gardner's syndrome
C. Turners hypoplasia
60 most common organism which causes caries
A. Lactobacillus
B. Streptococcus mutans
C. Actinomycetes
61.Anticariogenic sugar substitute is :
A. Xylitol
B. Mannitol
C. Sorbitol
62.Carbohydrate effect on caries by
A. Duration
B. Form
C. Type
D. frequency
63.pt.has upper right 6 endodonticaly treated tooth with small MOD caries.The best treatment:
A. Gold crown
B. MOD gold inlay
C. MOD gold onlay
64 Dental implant are successfully with minimum failure:
A. pre maxilla area
B. post.area of maxillary arch
C. mandible between mental foramen
D. buccal shelf of the mandible
65.antibiotics prescription is recommended when:
A. diffuse rapid spreading infection
B. acute localized infection
C. chronic infection
66.Treatment of acute pericoronitis?
A. antibiotic only
B. Reduce the opposing tooth and sub gingival
C. rinse with antiseptic
D. no treatment
67.After serial extraction what type of sutures??
A. fig 8
B. interrupted
C. horizontal mattress
D. vertical mattress
68 somewhat same pic of tori with long question
69.Which of the following statement is true regarding dental calculus:
A. It is composed entirely of inorganic material.
B. It is mineralized dental plaque.
70.dentist at the end of the day want to pour alginate imp quickly how can he do
that
A. Increase powder/water ratio
B. Hot water
C. Slurry water
D. Increase thickness
71.(pic was there.but I couldn’t find exact pic in google)fracture of upper denture in midline from
ant to pps and patient said that it
was broken before 2 or 3 times n by examination presence of
inflammation on residual ridge n sever bone resorption Wat
causes the fracture
A. Ill fitting denture n thick frenum
B. Unbalanced occlusion
C. Thin denture
72.the extracanal is present in which root of 1st max. Molar
A. palatal root
B. mesiobuccal root
C. distobuccal root
D. mesiopalatal root
73.42. the use of low speed hand piece in removal of soft caries in children is better than high
speed because
A. less vibration
B. less pulp exposure.
C. better than high speed
74.1.After gingivectomy surface epithelisation occurs in
A. 3 days
B. 5-14
C. 14-21
D. Over a month
75.After trituration of amalgam condensation must be
A. after (3_4)min.at least in order to remove excess mercury
B. vertically
C. with little pressure
D. immediatly.
76.Sharp pain is due to which type of fibers?
A. A delta fibers
B. B delta fibers.
C. C delta fibers.
77.which laser can we use instead of halogen composit light
A. Co2
B. Nad yag
C. Argon
78.Device that used to detect fissure caries with electrical resistance
A. Laser
B. Fluorescence
C. Electric caries measurement
79.If you did two holes of rubber dam too close what will happen :
A. Difficulty of putting dam interdentally
B. Stretching of dam will happen and subsequent leakage
C. Wrinkling of dam
80.The depth of cavity prep. for composite in posterior:
A. Limited to enamel.
B. 0.5 mm. in dentin.
C. Depends on caries extension.
D. Depends on tooth discoloration.
E. 0.2 mm. in dentin.
81.whats the meaning of attachment level
A. from gingival margin to depth of pocket
B. from marginal groove mucogingival line
C. from dentino cementum junction to mucogingival line
D. From cemento enamel junction to mucogingival line
82.patient came with ulcer on the dorsum of tongue. Lab report says that Poorly differentiated
squamous cell carcinoma. What does that mean?
A. Good prognosis , high recurrence
B. Bad prognosis , high recurrence
C. Bad prognosis, low recurrence
D. Good prognosis , low recurrence
83.PRR indicated in
A. Shallow caries involving entire fussure
B. shallow caries involving half and other half deep
C. caries involving only half of fissure
84.Development of mandible from which cartilage lateral to pharyngeal arches
A. Meckel’s cartilage
B. Reichertz cartilage
C. Thyroid cartilage
85.time of curing in very small class 3 composite restoration
A. 15 sec
B. 20 sec
C. 10 sec
D. 5 sec
86.Decrease the effect of acid etching on the pulp the three length of filed and reamers that u
work by them :(question was entirely different)
A. 20_26_29
B. 21_25_32
C. 20_25_32
D. 21_25_31
87.A patient made for himself a complete denture. After a few days he comes to you complaining
from pain and white spots on the residual ridge and you do relief in that area and give him
ointment. After a few days he comes again complaining the same but in another area. The
main cause is:
A. Uneven pressure on the crest of alveolar ridge.
B. Rough tissue contacting surface of denture
C. Increase vertical dimension
D. Absence of balancing occlusion
88.Female come with mass on left neck, slow growing before 6 years, the first surgeon said it is a
harmless sialodenitis, now CT scan show mass on submandibular gland, your diagnosis:
A. sialodinitis
B. pleomorphic adenoma
C. adenoid cystic carcinoma
89 What is the goal of maintenance therapy
A. To prevent recurrance of disease
B. Evaluate tissue response
90.upper 8 Impaction to avoid tear of gingival flap
A. Adequate size of flap
B. Flap include greater palatine nerve
C. Strong retraction of flap margin
91.After bleaching, want to restore a tooth with composite. You don't want to compromise
bonding. How long should you wait?
A. 24hrs
B. A week
C. After 2 weeks
D. Another material for restoration
92.Increase in the chance of fracture of anterior teeth if?
A. Caries
B. Protrusive anterior
C. Weak enamel
93.scammons curve of growth-which attains the highest 1st??
A.Neural
B.Genital
C.Lymphoid
94.During which period of intra uterine life the primary tooth bud is formed:
A. 4weeks
B. 5weeks
C. 6 weeks

EXAM – 13

.
1.vazirani akinosi technique -trismus case
2.twice ianb given.failed.how to manage?-gow gates
3.lower lip malignancy which is most common?
a.Mec b.acinic cell ca c.acc d.low grade tumor
4.canine palatal impaction vs buccal impaction ratio
2:1
5.2mm plate drill hole size
1.5mm
6.pic of missing 11; 12 has mesial caries .pt needs restoration of 11.no mobility,
healthy perio,no other systemic disease. What is best diagnostic image for 12?
A.periapical b.bitewing c.panorama d.occlusal
7.warfarin inr on day of surgery
8.ideal amount flouride in water.
0.5 to 0.8
9.shade guide for cement during porcelain veneer restoration.
10.trauma case, ant teeth move as one segment diagnosis?
Alveolar bone#
11.vanderwoude syndrome -congenital lip pit
12osteogenesis imperfecta
13.prognosis after rct determined by
Decrease in size of radioluecency
14.main component of inlay casting wax
Paraffin wax
15.newly erupted (pic) teeth stained groove no catch best managed by?
A.Flouride n 6 month follow up
B.sealant
C.PRR
D.composite
16.which one gives good glossy finish after restoration?
Microfill composite
17.best radiograph for implant
CT , periapical, panoramic views, MRI
18.technique of anesthesia in hemophilia pt
Intraosseous
19. During upper 3rd molar extraction tooth pushed posteriorly and superiorly
unable to visualise, management? Admit Ct and extract under GA
20. Gingivectomy in nifidipine induced gingival enlargement, is done to,
Eliminate pseudo pocket
21. Shape of two rooted maxillary premolar access cavity?
Oval
22.maxillary premolar extraction forceps
23.uncontrolled diabetes carious 37 paresthesia of lower lip moth eaten
radiograph
Acute osteomyelitis
24.fractured subgingival curette removed with
Schwartz periotriever
25.class 5 restoration excess removed using
A.Knife B.carver c.
26.semi adjustable arcon articulator example
A.dentatus
B.denar 5 a
C.denar mark 2
D.gnathoscope
27.pseudo class 3 management?
A.retraction of upper
B. Retraction of lower
C. Lip bumper
D.
28.bone of nasal septum -vomer
29.mandible formed from?
Meckels cartilage
30.pt with upper n lower denture having anterior ridge with white ulcerations
came to dentist. Correction was done.patient was comfortable. Again came with
some problem in another area. Cause for lacerations?
Uneven pressure on ridge
31.pt.came for extraction on examination there's a white patch with ulceration
on buccal mucosa.pt is tobacco chewer.best management?
Referral to surgeon for biopsy
32.nerve supply of palatal mucosa of maxillary first premolar?
Greater palatine nerve
33.radioleucency in anterior mandibular region with vital teeth. Percussion and
cold test normal
Periapical cemental dysplasia
34.chronic maxillary sinusitis microbiology?
A.predominantly aerobic
B. predominantly anaerobic
C. Mixed aerobic n anaerobic
D.80% anaerobic 20%aerobic
35.motor supply of scalp.
Temporal br of facial
36.dis adv of full thickness flap.
Scar formation
37.cyst surrounding impacted 3rd molar
Dentigerous cyst
38.structure that oppose retentive arm in rpd
Reciprocal arm
39.caries progrssion in old pt vs young patient
Generalised sclerosing by age
40.acute pericoronitis management?
A. removal of flap
B.removal of flap n antibiotic
C.extration
D.
41.trauma.,apical root fracture with wide gap between fractured segment.
Management?
42.conservative method of tooth whitening?
Bleaching
43.function of indirect retainer?
Prevent movement away from tissue
44.side of the bristle actively participate in?
Modified still man
45.overhanging restoration mainly affect?
Periodontal health
46.child with lower canine abscess with no crowding best managed by?
Extract both canine without space maintainer
47.trauma case.missing 11.fracture of 12 fracture of buccal plate
.restoration using?
A. Acrylic rpd
B.tooth supported fpd
C. conventional fpd
D.metallic rpd
48.biologic width- base of the pocket to cej
49.wrought wire clasp advantage over cast wire clasp
Less irritation to abutment
50.student taking xray of lower molar in female patient induces gag.what is the
cause of gag reflex?
Soft palate and fauces
51. Single bilateral edentulous area anterior to remaining natural teeth
Kennedy class 4
52. Dis adv of akers clasp
Mobility
53.difoti advantage
Determine caries activity
54.perforation at root bifurcation. Material used?
MTA
55.mentally and physically disabled , pouching of food in cheek
Autism
56. In a denture lower 3rd molar set in occlusal plane.
A.post 1/3rd of retro molar pad
B. Ant2/3rd of retro molar pad
C.class 1
D.class2
57.curing light 450nm wavelength Wat is ur decision?
58.scarlet fever, white lesion.scraped.red bleeding area.diagnosis?
Candidiasis
59. Pt quit smoking develop aphthous. What is the cause?
Nicotine out of blood
60.2wall defect what is the best graft to treat this defect?
Cancellous FDBA
61.sjogrens syndrome

EXAM – 14

1. Hemophilia B - decrease in which factor? > Factor IX

2. Supply of upper 4 palatally by:


A. Lesser palatine
B. Greater palatine
C. Nasopalatine

3. Simplest way to stop bleeding?


A. Retraction cord after matrix band
B. Use cotton pellets press for some time

4. Most important for dentist to know?


How to manage situation and control

5. Mother called dentist, her child swallowed fluoride?


Give calcium supply and take to emergency directly

6. How to measure if periodontitis is healing?


Decrease bleeding on probing
I chose> decrease periodontal pocket

7. Decrease salivation after giving LA?


Options were about effect on para sympathetic…. and efferent….
I chose > decrease sensitivity in mouth to objects

8. All are preventive method except?


A. Pit and fissure
B. Fluoride tooth paste
C. Fluoride in water
D. GIC restorative

9. What’s the access opening for upper lateral? > oval

10. What is the disadvantage of one plane preparation?


Less retentive
11. X-ray of impacted 3rd molar, what is diagnosis?
A. Radicular cyst
B. Dentigerouse cyst

12. Long case at the end written multilocular giant cell, what is
diagnosis?
Giant cell granuloma

13. Read about spaces in children (leeway, primate space)


Photo of child upper teeth with arrow asking what this space is? (It was
between canine and premolar)
A. Physical space
B. Leeway space
C. Primate space

14. What connect the abutment to prosthesis?


A. Pontic
B. Rest

15. X- ray of lower 6 with radiopacity in root area (around root),


diagnosis?
A. Hypercementosis
B. Condensing osteitis
16. Forceps to hold tissue while suturing? > Addison’s forceps

17. Question about scaling device used? Ichise ultrasonic scaler

18. Sub gingival scaler? > Gracey

19. Instrument to measure furcation? > Nabers

20. Irreversible hydrocolloid (alginate) impression left 15 mints then


poured with stone, cast was milky and chalky, Cause?
I chose dehydration of impression

21. Patient came from another clinic with perforation? > use MTA

22. Why to use calcium hydroxide between visits? > Because of


antimicrobial effect

23. A process to remove half of tooth (MD or Buccal- lingual) and keep
the other half in bone to be restored?
A. Hemi section
B. Amputation done

24. Type of graft from same person? > Auto graft

25. GIC advantage? > release fluoride

26. Question about amalgam? > Convergence occlusally

27. Submandibular duct opens where?


A. Frenulum of tongue
B. Floor of mouth
28. over contoured crown?
Accumulate plaque

29. Most surface collect plaque? > Upper buccal posterior teeth

30. Photo of pterygomandibular raphe, what is this type of anesthesia?


A. Gow gate
B. Alveolar block

31. 3 x-ray with small radiolucent circles what’s the diagnosis?


A. Osteosarcoma
B. Ameloblastoma

32. Why to advice pt to brush his teeth? > To remove food debris

33. Pt with short RCT, retreatment, GP removed, dentist can’t locate the
apex in the canal, why?
Forgotten the options!!

34. Post preparation of upper 4, perforation occurs what is the reason?


I chose furcation is more cervical (most reasonable option)

35. Patient with class 2 composite restoration came after two days
complain of pain why?
Overhanging
Forgotten the other options!!

36. Severely fractured tooth due to caries lesion, tooth is vital what is
the best to give retention and resistance?
A. Crown with post
B. Amalgam bond ( i chose )

37. What’s the fatal dose of fluoride?


A. 345 micro
B. 400 micro
C. 5-10 micro ( i chose )
D. 1 gram

38. Child swallows fluoride what’s the dose and time to be injected?
Forgotten the options!!

39. Photo of child with lesion white spots in half of palate from posterior
teeth to anterior including lateral and central, what is the diagnosis?

40. Old patient with denture with poor oral hygiene complaining of pain?
Denture stomatitis

41. Cross section of wrought wire? > Round

42. Anterior posterior strap used in?


I chose u shape (SLE)

43. Patient with caries in lower posterior you found there is sever
attrition, best management?
A. Composite restorations
B. Crowns ( i chose )

EXAM – 15

1. best xray for proximal teeth ?

2. 7 years old pt came with fracture in subcondylar area.


the other side have class 1 malocclusion , fracture side have class
2 malocclusion . manual reposition it shows class 1 . whats you
tt? A. open reduction . b closed reduction. c . follow up .

3. instrument with number 15 . 83. 8. 14 . which is for length of


balde ?

4 . 3 years under general anasthesia , xray show small caries


proximal area D and E , tt ?
A. composite
B. amalgam
C.stainless crown
D. pulpotomy with stainless

5.convergence of walls amalgam

6 . pt 17 years started forming open bite , no sucking finger habit


.
A. cleidocranial dysplasia
B. eagle syndrom
C. treacher collins
D .plummer vinson

7. pt had radiotherapy 4 years back . edentulous, need CD .


which material to use for impression
A. compound
B. plaster paris
C.zoe

8. standard and conventional gp difference

9. space between 2 implant

10. non resorbable suture , most used in oral surgery .

11. pic of fibroma

12. GP : 70 % ZOE 30% gp

13. why use low speed bur for temporary teeth ?


14. dentist romoves all caries in a tooth , than saw a small red point
bleeding lighly .
A . direct pulp capping
B. indirect
C. pulpotomy

15. Percentage of copper in amalgam which reduces gamma 2


phase?

16. crucial need for appliance to stop thumb socking


A. temporary dentition
B. early mixed dentition
C. late mixed
D. permanent dentition

17.
diabetic patient suffering 15 days came with swelling , erythema ,
pain in the area of right mandibular molars ( no mobility) . xray ;
moth eaten appearance . diagnosis ?
which osteomyelitis?

18. oxygen flow rate per minute .

19. 7 years old , intruded max ant teeth after fall . tt

20. Pt have calculus and u want remove using device have


elleptical motion .. Which is it...?
A.ultrasonic
B.piezo
C.hand instrument
D.magnetic

21. Brush method with side parts of bristle activated ?

22. During 3/4th crown preparation on pm bur used to add


retentive grooves

23. character of irreversible pulpitis ?

24. Most potent vasodilator?


a.cocaine
b.procaine
c.tetracaine
d.atricaine

25. The substance in local anaesthesia cartridge responsible for


prevent oxidation of vasoconstrictor: A.sodium chloride solution
B.sodium metasalphate C.sodium salphate

26. bionator

27.When you give sedative inhalation for patient to prevent


hypoxia u give :
A-95% oxygen and 5% nitrouse oxide
B-90% oxygen and 10%nitrous oxide
C-85%oxygen and 15%nitrous oxide
D-100% oxygen and zero nitrous oxide

28.patient came to hospital with gun shot ,,the surgeon will make
fixation by : a- christian's technique b- keen's technique

29. Pt. got gun shot , question about graft used for condyle ?

30. best xray for parotid

31. swelling when eating . sialothitis

32. second best xray for planning and fixing implant

33. 2 layers of varnish under amalgam .

34.class 1 malocclusion , the vertical dimension


A. 1 mm less in rest position
B. 5 mm
C. 6mm

34. which part of periodontal tissue regenerate the last ?

EXAM – 16

1. pt. with white spot on his tooth and tooth not cavitated tell ptt to check
up after?
A. 3 months
B. 4-6 months
C. 7-9 months
D. 11-12 months
B????
2.Distance between patient and cephalometric
5 feet *****

3.patient with anaphylactic shock due to pencillin


1- 0.5-1epinephren of 1\10000 Im
2- 0.5-1 1\1000 adrenalin im ******
3- 200 mg hydrocortisone intravenous
4- Other option not related

4.patient came to clinic with wrinkled skin and white shiny hair, with
pegged laterals teeth :
A) ectodermal dysplasia******
B) cleidocranial dysplasia
C) pteygz jogheurs syndrome
D) Gardner syndrome
5.Which material used in one-visit pulpectomy:
a. MTA (Mineral Trioxide Aggregate)***
b. Ca (OH) 2 + CMCP
C. Formocresol
D. Zn oxide

6.Critical ph of saliva at which demineralization of enamel begins :


A-5--5.5 ***
B-4---4.5
C-6---6.5

7. best describes caries progression?


a. Dynamic process with demineralization and remineralization***
Other 2 option I can’t recall

8.Color choice for cervical third for full coverage


A-highest chroma****
B-thick enamel
C-highest value
D-lowest hue

9.The concentration of household bleach is:


5.25%***

10.If tooth or root is pushed during surgical extraction into max. sinus:
a. Leave it and inform the Pt.
b. Remove it as soon as possible. ***

11.Post graduated student usesMTA the prognosis depends on prevent:


a. immediate suture.
b. disturbance during closure of wound. ***
c. using a flab

12.8 yrs old pt has lower primary canine space infection what is the ttt?
1. Extract one canine without space maintainer
2. Extarct canine and band and loop
3. Extract both canines without space maintainer
4. Extract booth canines with lingual arch holding bar

13.to make v shaped groove in canine what bur should be used according
to some standard:
a)round
b)tapered
c)fissure
d)inverted cone

14. Patient came to clinic with wrinkled skin and white shiny hair, with
pegged laterals teeth :
A) ectodermal dysplasia****
B) cleidocranial dysplasia
C) pteygz jogheurs syndrome
D) Gardner syndrome

15.Saturation :
A chroma***
B hue
C value

16.Dentinogenesis imperfecta occurs in which stage.


A.histodifferntiation***
B.morphodiffrentiation
c.apposition

17.After ortho pain upper canine , x ray resorption canine root


a. Apply CaoH at the site of resorption***
b. Do RCT in a single visit.
c. Extract the tooth & reimplant it.
d. Extract the tooth & do implantation.

18.
Pt. come with severe pain, no response to pulp test when you do
percussion
the patient jump, diagnosis is: (No periapical change in radiograph
)
1.symptomatic reversible pulpitis .
2. Symptomatic irreversible pulpitis .
3. Asymptomatic apical periodontitis and asymptomatic irreversible
pulpitis. **

19.hiv posatavaly test


a. elisa *****
b. westron blot
Other 2 option I forgot

20.Access opening of lower canine is


Triangle
Square
Ovale*****
Trapizoidal

21.water irrigation device:


1. Prevent plaque formation
2. Completely removes plaque
3. Dilute bacterial toxins***

22.By radiometry u found the halogen light cure is 450 what u will do
a. change battery
b. change the lamb
c. don't make Any change.****
23. pt with missing 4 ant teeth and need fpd what abutment you choose
a. 2 canines
b. right canine and left canine and premolar
c. left canine and right canine and premolar
d. 2canines and 2 premolars

24.A tooth with fracture cusp dentine involves what is the status of pulp
in this case?
a. un inflamed healthy pulp
b reversibly inflamed
c. irreversibly inflamed
d. innervated with A delta fiber

25.best material for impression of flappy ridge :


a- plaster of Paris*****
b- ZnOE
c- agar agar
d- compound
26.Active ingredient of Hemodent:
a. Ferric sulphate
b. Zinc phosphate
c. aluminum chloride****
d. ferric chloride

27.the CBCT have the following property


a- best to show TMJ disk***
b- expose the patient to large amount of x-ray
c- use for routine radiographic examination

28.access cavity for lower 2nd molar:


1-triangular with base towards mesial******
2-triangular with base towards distal
3-oval with base towards mesial
4-oval with base towards distal

29.A root seldom has 2 pulp canals


A. mesiobuccal root of upper molar
B. mesiobuccal of lower molar
C. distobuccal root of upper molar******

30.The organism that rarely found in newborn mouth:

a. Streptococcus mutant. ***


b. Streptococcus salivarius.
c. Spirochaeta.
d. e-coil.
e. Skin bacteria.

31. Fluoride water supplementary and we want to give systemic fluroide


what to ask pt
A.Age*****
32. The common concentration of Fluoride in over the counter
dentifrices in ppm is?

1000 ppm****

33. Cavity etching before applying GIC is:


1. Polyacrylic acid 10 seconds.****
2. Polyacrylic acid 60 seconds.
3. Phosphoric acid 10 seconds.
4. Phosphoric acid 60 seconds

34. Patient had bulimia and had lesion in palatal surface in upper teeth
with recurrent vomiting. What is the type of lesion
1. attrition
2. abrasion
3. erosion*****

35. according to Kennedy class of bilateraly single edentulous space


found anteriorly
class 4 ****
36. treatment oral and pharynx candida.
Fluconazol***

37.Which transformation of oral mucosa should be taken seriously


dysplasia***
Metaplasia
hyperplasia
neoplasia

38. CATAR:
1.One wall defect
2. Two***
3.Three

39. PT. Who has iron deficiency anemia difficulty in swallowing with
examination of barium sulphate.
A. Geographic tongue.
B. Burning mouth syndrome.
C.plummer vinson syndrome. ****
D.diabetic patient.

40. time in days to established gingivitis


14-21 days****

41. colour of complex seen in a 20 year old healthy periodontium


Black complex
Purple complex******
orange complex
red complex

42.Long case of removal of epilis fissuratum. What forcep you use


- stillies forcep
-Adson forcep
-Allis forcep******

43.Best implant material


Titanium*****
Platinum
palladium

45.Brushing technique Active side of bristles


Modified stillman****
Bass
Charters

46. Needle for vazirani akinosi technique


25 gauge long needle***
47. Continuous condensation technique in gp filling is:
a. obtura I.
b. obtura II.
c. ultrafill.
d. System B.*****

EXAM – 17

1. After infection with chicken pox, Isolation period should be:


a. When fever subsides
b. After one week
c. When the vesicles are crusted****
d. Until carter stage is last
2. Primary role of epinephrine of local anesthesia during apical surgery:
a. Reduce systemic absorption so reduce toxicity****
b. Increase duration
c. Two other options were about alpha & beta receptors, I can’t
remember
3. Streptococcus Mutans affects which tooth surfaces:
a. pits and fissures****
b. smooth surfaces
c. root surface
4. Most common intracanal medicament:
a. CaOH****
b. Formocresol
5. Angle of blade while scaling:
a. 45-90****
b. 70-80
c. 100-110
6. To disinfect impression material, use:
a. Formaldehyde
b. Acetaldehyde
c. Gluteraldehyde****
7. Amalgam free of gamma 2 is:
a. 2% cu
b. 5% cu
c. 10% cu
d. 13% cu****
8. What's most difficult in impaction surgery
a. Mesio angular
b. Disto angular****
c. Horizontal
d. Vertical
9. After removal of impacted 3rd lower molar, pt complaining from
parasthesia, why?
a. Irritation of the mandibular nerve during extraction****
b. Broke mandible
c. Hemorrhage from the socket
10. The instrument use in suturing of 3rd molar:
a. Stillis forceps****
b. Addison forceps
11. Brushing technique Active side of bristles
a. Modified stillman****
b. Bass
c. Charters
12. Which material is used in one-visit pulpectomy:
a. MTA (Mineral Trioxide Aggregate)****
b. Ca (OH) 2 + CMCP
c. Formocresol
d. Zn oxide
13. Patient had bulimia and had lesion in palatal surface in upper teeth
with recurrent vomiting. What is the type of lesion:
a. Attrition
b. Abrasion
c. Erosion****
14. Conventional GI cement has an advantage comparing to other GI
types:
a. Fast Setting
b. Shelf life
c. Strength
d. F concentration
15. The three length of file and reamers that you work by them:
a. 20_26_29
b. 21_25_32
c. 20_25_32
d. 21_25_31****
16. Mucocele is treated by :
a. Marsupalization
b. Excision****
17. Restoration that is contraindicated in mouth breather pt:
a. GIC****
b. Compomer
c. Direct composite restoration
d. Indirect composite restoration
18. Amalgam used in retrograde filling is:
a. High copper
b. Zinc free****
c. Mercury free
d. Antibiotic treated
19. Pt came to clinic with avulsed central incisor, 30 min ago, stored in
milk, PDL was good, splint for how many weeks:
a. One
b. Two****
c. Three
d. Four
20. Proxy brush is used with which type of embrasure:
a. Type 1
b. Type 2****
c. Type 3
d. Type 4
21. After GTR (guided tissue regeneration) what type of dressing should
be used:
a. Eugenol dressing
b. Non eugenol dressing****
c. Antibiotic dressing
22. First sign to show if there's lidocaine toxicity:
a. Bradycardia
b. Tachycardia****
c. Cardiac fibrillation
23. A disease occur in 350 persons out of 1000, the prevalence of the
disease in 7 years will be:
a. 35
b. 50
c. 380
d. 2450****
24. PDI classification for missing teeth in upper and lower arch including
canines:
a. cl1
b. cl2****
c. cl3
d. cl4
25. Dental material classification:
a. Ceramics, polymers and composite
b. Ceramics, polymers and alginate.
c. Ceramics, polymers and cement
d. Ceramics, metals, polymers, composites****
26. Bacteria for infective endocarditic:
a. S. aureus ****
b. S. virridans
c. Actinomyces
27. Most used as anticariogenic:
Xylotol****
28. Arrange the steps of cleft palate management:
a. Measures to adjust speech.
b. Establish way for nursing and feeding.
c. Cosmetic closure.
d. Prevent collapse of two halves.
I forgot the options, but I think the order is B < D < A < C
29. First line treatment of 3 years old child with oral candidiasis:
a. Nystatin oral suspension****
b. Fluconazole systemic
c. Ketoconazole systemic
30. Serial extraction for rapid eruption in patient 11 years:
a. Mandibular permanent first premolar****
b. Mandibular permanent second premolar
c. Deciduous canine
d. Deciduous first premolar
31. (Pic of floating teeth) pt with mental confusion, renal calculi, and high
alkaline phosphatase:
a. Hyperthyroidism
b. Hyperparathyroidism****
c. Hypoparathyroidism
d. Hypothyroidism
32. Pt has facial asymmetry, what type of x-ray used:
a. Anterior posterior object
b. Orthotomograthy
c. CBCT
33. Which is easiest to stop gingival bleeding in class II cavity
preparation:
a. Electrical cautery
b. Retraction cord after matrix band
c. Pressure with wet cotton pallet****
34. Local anesthesia technique used to block the buccal, lingual and
mylohyoid nerves:
a. Gow-Gates Technique****
b. Vazirani-Akinosi Technique
c. Coronoid approach
d. IAN Block
35. Why acrylic teeth are used NOT porcelain in dentures:
a. Acrylic resist staining more
b. Acrylic bond to denture base more ****
c. Acrylic have high compressive strength
d. Acrylic resist wearing more than porcelain
36. 25 years old pregnant patient has bleeding on probing on papilla of
anterior area of the maxilla:
a. Giant cell granuloma
b. Pyogenic granuloma****
37. Bacterial spores used as a test for autoclave, it is considered:
a. Chemical test
b. Physical test
c. Biological test****
d. Sterilization test
38. What is not a denture surface?
a. Polished surface
b. Vestibular surface****
c. Occlusal surface
d. Impression surface
39. Pt 9 years old, Apexogenesis is done to his upper central incisor,
when to judge that the treatment done is successful:
a. Root complete development
b. Tooth is asymptomatic****
c. Tooth responds normally to vitality test
d. Inflammation confined to the coronal pulp
40. Pt has necrotic pulp in upper central incisor, not closed apex, what
is the best treatment:
a. Calcium hydroxide
b. Apexification with GP filling
c. Calcific barrier ****
d. RCT + GP filling
41. Most suitable to describe NITI:
a. Rigidity
b. Low coefficient of friction
c. Shape memory****
42. Bacteria for initiation and progression of caries: << Streptococcus
and lactubacillus
43. Advantage of GIC: << Release fluoride
44. Bacteria cause endo failure: << E. feacalis
45. Pterygopalatine ganglion is anatomically related to which nerve:
a. Maxillary nerve ****
b. Facial nerve
46. Which fluoride can be applied professionally by dentist:
a. Act
b. Duraphat****
c. Gel tin
d. Prevident
47. .Interproximal bone parallel to:
a. Gingival margin
b. CEJ
c. PDL****
d. DEJ
48. Extraction after dialysis
a. 1 hr after hemo dialysis
b. 1 day before ****
c. 1 day after
d. 1 week after
49. Type of cementum in coronal 2/3rd:
a. Acellular afibrillar
b. Acellular extrinsic fibre****
c. Acellular intrinsic fibers
d. Cellular mixed stratified
50. Difference between PRR and class I filling:
a. Caries extension****
b. Caries depth
51. Best location and size of root perforation that is favorable:
a. Small perforation below height of bone crest
b. Large perforation below height of bone crest
c. Small perforation at height of bone crest****
d. Large perforation at height of bone crest
52. Pt complaining from bleeding while brushing, on examination,
swollen red gingival, shiny and loss of stibling, bleeding on probing with
3-4mm probing depth without attachment lost:
a. Acute gingivitis****
b. Chronic gingivitis
c. Periodontitis
d. ANUG
53. The access opening for upper central and lateral incisor is:
a. Triangular****
b. Rectangular
c. Square
d. Oval
54. The most used material for home bleaching:
a. Hydrogen peroxide****
b. Sodium perborate
55. Food low cariogenic potential, the following should be characteristic
a. Low buffering capacity
b. Ph higher than 3
c. Contain mineral****
d. Contain protein
56. Pt with denture complains of poor retention, when you press on
palate you see bubbles in posterior extension due to:
a. Over extension
b. Over post damming
c. Under post damming****
d. Under extension of post dam
57. When to select the composite shade for restoration:
a. Before rubber dam placement****
b. After cavity
c. After bonding
58. Sharpening of curette, you put its cutting edge to stone at which
angle:
a. 70-80
b. 80-90
c. 60-70
d. 100- 110****
59. Mechanism of mandible growth:
a. Apposition and intra membranous modeling
b. Interstitial and endochondral
c. Apposition and endochondral ****
d. Interstitial and intra membranous
60. Which deciduous tooth cause crowding in lower anterior region if
early extracted:
a. Primary mandibular first molar
b. Primary mandibular second molar****
c. Primary maxillary first molar
d. Primary maxillary second molar
61. Pt. needs complete denture, you take impression with irreversible
hydrocolloid (alginate) & poured it after more than 15 min. the cast
appears soft & chalky the reason is:
a. Dehydration of the impression****
b. Expansion of the impression
c. Immerse the impression in disinfectant
62. Measuring pulp vitality:
a. Heat****
b. Ozone O3
c. Carbon dioxide
d. Laser Doppler
63. Dye that is used with toludene blue to differentiate between cancer
and normal cells
a. Methylene blue
b. Congo red
c. Lugol****
64. Pt with painless, firm, bilateral enlargement on lower mandibular
region, on x-ray, multilocular radiolucency with displacement of the
lower 2nd molar. What is the diagnosis:
a. Crhon’s Disease
b. Behçet's Disease
c. Odontogenic Keratocyst
d. Cherubism
65. Pt came to the clinic after 3 days of composite restoration was done,
complaining from color was changed and not matching the adjacent
teeth. What you will do:
a. NO treatment
b. Add layer of composite
c. Apply bonding agent
d. Resurfacing ( I don’t remember the option)
66. Success rate of implants
a. 85% to 94%
b. 95%
c. more than 95%
67. Pic of teeth with erosion
68. Pic of patient wearing CD, what was the problem?.. I chose mid line
shift
69. Pic of intraoral lesion, the options were:
a. Fibroma
b. Epilus Fissuratum
c. Flappy Ridge
d. Papillary Hyperplasia
70. Q about mesio-buccal cusp crack in upper molar, what’s your
management?.. Sorry I forgot the options

71. Q about how new technology/methods helped in detecting caries?


72. Pt came to clinic with multiple missing (non-erupted) permanent
teeth, what type of x-ray you will take?

EXAM – 18

1.L.A. given to pregnant lady.


a)procaine
b)Prilocaine
C) lidocaine ***
2.pic of facebow.type of facebow
a)condyle
b)ear
C)kinetimic*****
3.dental floss used:
a)remove calculus
b) remove overhang
c)disrupt the plaque
4.During working on a patient you noticed a furcation perforation,what
is the best material to manage this;
a)calcium hydroxide
b)MTA****
5.Length of the needle when injected to the mucosa in Inferior Alveolar
block:
a)two-third of the needle****
b)one-third of the needle
6.Technique will you use to anesthetize soft and hard tissuse of
mandibular with one injection:
a)Inferior Alveolar nerve block
b)Gowgates*****
c)vazirani akinosi
7.pic supernumerary teeth
8. After finish class V glass ionomer cement we do finishing with:
a) Pumice slurry.
b)Aluminum-oxide disc.*****
c)Prophy paste with fluoride
9.In acid etching for fissure sealant:
a)Longer time for permanent teeth
b)Longer time for primary teeth****
c)Same for both primary and permanent
d)Longer in amelogensis imperfect
10.Hemophilia A child has tooth 58 resorbed but distal root not premolar
tooth cusp tip has been showing patient has discomfort from tooth
58.what is the treatment:
a)extraction
b)no intervation****
c)pulpotomy
11.What is the copper ratio that eliminates gamma phase 2:
a)13%****
b)10%
c)2%
12. Cavity etching before applying GIC is :
a) Phosphoric acid 10 seconds.
b ) Polyacrylic acid 60 seconds.
c ) Polyacrylic acid 10 seconds.********
d ) Phosphoric acid 60 seconds.
13.Patient with missing upper 12 and 21.what kennedy classification:
a)class 1 modification 1
b)class 2 modification 1
c)class 3 modification 1
d)class 4 modifiction 1
14.When all the teeth is missing except the two canine,according to
kennedy classification is:
a)class 1
b)class 2
c)class 3****
15.PDI classification for missing teeth in upper and lower arch including
canines:
a)class 1
b)class 2*****
c)class 3
d)class 4
16.During composite filling MO in premolar there was overhang
restoration to avoid this:
a)semi sectional matric with wedge*****
b) full tofflemire matric with wedge
c)celluloid matric
17. Caries susceptible surface in a high caries patient
a)Facial surfaces of maxillary posteriors****
b)Facial surfaces of max anteriors
c)Palatal surfaces of max posteriors
d)Palatal surfaces of max anteriors
18.In recent DIAGNODENT is used for detection for specifically
diagnosis that cannot be done by another device(something like this):
a)proximal
b)deep
c)superifical
d)bacteria invasion
19. -Material used in oral cavity binds to all surface and has ability to
release ions and decrease microbial effect:

a)Flouride***
b) chlorhexidine mouthwash
20.an old patient came to clinic completely edentulous need to make
complete denture what is xray of choice:
a)bitewing
b)periapical
c)panomaric radiograph****
d)CBCT
21.to make v shaped groove in canine what bur should be used
according to some standard:
a)round
b)tapered
c)fissure
d)inverted cone
22.sensory innveration to orofacial:
a)facial N
b)occulumotor N
c)abducens N
d)trigeminal N
23. While performing cranial nerve examination you notice that the
patient is unable to raise his eyebrows, hold eyelids closed,
symmetrically smile or evert his lower lip. This may indicate:
a. Trigeminal nerve problem.
b. Facial nerve problem.****
c. Oculomotor nerve problem.
d. Trochlear nerve problem.

24.Desired resorption in ortho tooth movement:


a)undermine resorption
b)alveolar resorption
c)hyalination
d)facial resorption

25. pt. came to dental clinic for extraction.after extraction


bleeding profusely when check haemological problem in lab test they
found that factor VIII ( 8 ) is less 10 % what’s the diagnosis:
a- Hemophilia A.****
b- Hemophilia B.

26. 12 yr old child bleed easily with minor trauma have high bleeding
sight elevated time coagulation and fragile capillary he suspected to
have
a)Hemophilia
b)Thrombocytopenia****
c)Vit.k. deficiency
27. Female come with mass on left neck, slow growing before 6 years,
the first surgeon said it is a harmless sialodenitis, now CT scan show
mass on submandibular gland, your diagnosis:
A. sialodinitis
B. pleomorphic adenoma****
C. adenoid cystic carcinoma.
28.incidence infected with ceratin disease 350 in 1000 population.what
will be prevalence after seven years:
a)35
b)50
c)350
d)2,450***
29. Any attribute, characteristic or exposure of an individual that
increases the likelihood of developing a disease or injury.
a)confounding factor
b)prognostic
c)etiological
d)risk factor***

30. A female patient came to your clinic with dry lips and mouth and
bilateral submandibular oedema and ocular dryness. Diagnosis is:
a) Polymorphic adenoma.
b) sialotitis
c.)salivary stone
d) Sjogren's syndrome****
31. During Extraction tooth fall in patient mouth retrieve it with
a) college pliers
b) Russian forceps****
32. pituitary gland: growth hormone
33. Separate the tooth from the middle of molar and preserve the tooth
as two half premolar this procedure called:
A-tooth Hemisection
B- tooth Bisection****
34.tooth has radicular cyst what will be the state of pulp:
a)vital tooth
b)caries tooth
c)non-vital tooth******
35.few week after FPD pt complain of bad odour no pain what could be
the cause:
a)loosing retainer
b)open margin
c)food accumulation in between retainer

36.The important property of cement under amalgam :


a) High modulus of elasticity ***
b) Low modulus of elasticity
c) modulus of elasticity not important
d) High modulus of elasticity and low tensile strength
37. With long span FPD, unit should have:
a) Low strength
b) Low rigidity
C) High strength
D) High compressive and high rigidity***
38. old patient has discomfort in premolar , in x-ray there is abrupt
(sudden) midway canal dissapear , why :
a) secondory dentin apically
b) hypertropic calcification in apical part
c) bifurcation apical***
d) disorganised mass at middle
39.minimum dose of adrenaline for pt with anti-depressant drug:
a)0.01-0.02****
b)0.02-0.03
c)0.03-0.04
40.remants of rest of serres:
a)dental lamina*****
b)hertwiz sheet
c)vestibular lamina
41.water is interfered during direct restoration what will happen:
a)soft resin
b)decrease space for restoration
c)change colour
d)reduce streghth of restoration

42. smear layer benefit in protecting pulp through


a)prevent toxins to reach pulp through dentinal tubules****
b)decrease acid etch effect on pulp
43.h file superior than k file:
a)positive rank angle****
b)negative rank angle
c)more flutes
d)more in diameter
44.disinfectant used in home as home bleaching:
a)NAOCL****
b)lactic acid
c)alcohol
45.nine year old child extraction of maxillary right maxillary 1st molar
and left maxillary 2nd molar what space maintainer can be used:
a)nance
b)transeptal
c)hawleys applicance with thin wire
d)no treatment, reassurances
46.most retention of palatal form when applied with retentive agent:
a)u shaped****
b)v shaped
c)flat shaped
47. elderly patient with flat ridge & uncontrolled movement, best teeth to
use are:
a) 0 % cusp angulations "flat teeth" ***
b)10%
c)20 %
d)30%
48. Caries progression in adult less than child:
a)Difference in PH.
b) Generalized dentine sclerosis by age***
c) Increasing in organic content of tubular dentine by age
49.no caries on maxillary tooth will pressure pain radiates to ear no
radiographic finding:
Maxillary sinus
50.minimum space fo erupting of primary teeth for permanent teeth:
a)3mm
b)4mm
c)6mm***
d)8mm
51. aim of most periodontal surgeries:
a)Provide direct access and visualization to diseased root ***
b)Eliminate pockets
c)Lengthening gingiva
52.shape of rest seat in RPD:
a)spoon shape and square
b)concave****
c)convex
d)all
53.access opening for mandibular 1st molar:
a)rhomboid
b)base to buccal and triangle shape
c)trapezoid
54.base of the caries:
a)dentin
b)pulp
c)enamel
c)DEJ***
55.pic after orthodontic treatment there are white spots what is not in
option for treatment:
a)veneer
b)full crown****
c)parital veener
d)composite
56.minimum conservative whitening of vital teeth:
a)bleaching
b)microbrush****
c)veener
57.avulsed tooth best medium:
a)saliva
b)water
c)HBSS*****
58. Gingivectomy contraindication in :
a)massive alveolar bone surgery ***
b) Gingival inflammation
c)gingival abscess
59. the L.A depends on
a)strength bond between drug and nerve***
b) strength bond between drug and it's intensity
c) bind between drug and time of removal from body
60. cancer related to gardners syndrome in which organ:
a.colon***
b.lung
c.heart
d.pancreas
61. Caries detection dye composed mainly by
a)Acid fuschin
b)Basic fuschin
c)Propylene glycol****
62. Remove thick epulis fissuratum:
a) Allis forcep***
b) Addison forcep
c) Curved hemostat
d) Stilli forcep
63. Internal resorption in maxillary central incisor can be treated with
a)Ca(OH)2***
b) Gutta percha single sitting
c) ZOE
d) Silver cone
64. 2% taper of endo files means:
a) the difference from tip to D16. ***
b) 0.02 mm increase from the tip to the handle.
65. At the begining of the operation day in the clinic, you should start
the
water/air spray for three minutes in order to get rid of which type of
microorganisms :
a) Streptococcus mutans.
b) Staphylococcus
c)pseudomonas aurignossa****
66. The subgingival scaler to be specific area:
a) universal
b) The head should be 90% with shank
c)gracey curette****
67.function of rest in RPD:
a)support*****
b)retention
c)stability
68.antibotics prescribed for:
a)diffuse swelling***
b)periapiacal cyst
69.drugs cause ginigival enlargement:
a)phenyntoin****
b)cyclosporine
c)calcium channel blockers
70.Aker’s clasp used in RPD causes:
a)caries***
b)mobile
c)ginigival recession
71.before starting endo treatment what test should be done:
a)cold test*****
b)hot test
c)percussion
d)electric test
72.advantage of indirect over direct restoration:
a)occlusal interference
b)marginal adaptation***
c)high metal strength
73.onset of herpes simplex viruse without any treatment:
a)average 9-11 years******
b)above 10years
c)11-15 years
74.minimum time of topical LA applied:
a)30min
b)1min****
c)2min
d)3min
75. Theoretically which Xray technique gives exact length of root canal
:
a)Bisecting angle
b)PAralleling technique****
76.needle used for IAN block:
a)25gauge long needle******
b)25gauge short needle
77.young patient with multiple caries high plaque index erythema and
edematous ginigiva,ginigiva pocket 2-4.diagnosis:
a)ginigivitis****
b)junvile periodontitis
c)periodontitis

EXAM – 19

Iron deficiency anemia type : Microlytic


Pain form zinc-amalgam restoration : delay expansion
Raspberry on the palate : hyperplasia
Loss of gag reflex : IX
Impression of onlay : silicone
Hp B test is for : Immunity
Fiberoptic transillumination for incipient caries : all surfaces
Radiology pix of : odontoma, compound odontoma, ameloblastoma
Disease that no need for special care and not contagious : chickenpox (
rest options were Hp c and congectivitis and measles )
Patient with missing pri 1st molar on the left and missing canine and 1st
molar on the right ( lingual arch )
Needle holder : 1/3 of the needle
Restoration of p&f sealant : flowable

Questions am not sure abt the answers

Which disease of special care patient lead to periodontal destruction ( i


answered with down syndrome )
Space needed for eruption of pri teeth with no crowding ( 3-4-6-8)
Trauma Intrusion of pri centrals and effected permanent buds Rx ( carful
exo- observation- ectomy)

EXAM – 20

When you give a child a gift for a good behavior this is called:
A.Positive reinforcement.**********************
B.Negative reinforcement.

2-Optima water fluoridation:


a. 0.5-0.8 mg\liter.********
b. 0.2-0.5 mg\liter.
c. 2-3 mg\liter
d. 1-5 mg\liter

3-dye that is used with toludene blue to differentiate between cancer and
normal cells ( shaheen)

a. methylene blue
b. congo red
c.lugol***********

4-For recording of vertical dimention we use:

A.Willis Gauge.*****************
B.caliper.
C.Face bow

5-Avulsed tooth:

A.splint 1-2 weeks. **********************************************


B.splint 2-3weeks.
C.splint 3-4weeks.

6-bizygomatic width measured by:

A.gothic arch
B.facebow********************************
C.willis gauge

7-best feature of sealant?


A.viscosity
B.retention**********************

(sealant(pedo) >>> retention and sealer(endo) >>>viscosity)

8-Patient with hypertension with 140/100 and he has been using


medicine for the past 10 years he wants to make a denture , what do you
see ?

a.mucosal change
b.dry mouth*******************
c.gingival enlargement
d.fungal infection.

9-Dentin contains which type of collagen fiber ( shaheen )

a.Type I***************
b.Type II
c.Type III
d.Type IV

10-What is the copper ratio that eliminates gamma phase 2:


a. 2% copper
b. 4% copper
c. 10 % copper
d. 13 % copper********

11-Difference between amelogenesis imperfecta and dentinogenesis


imperfect

a.heridiatery factor.
b.brown color of enamel.
c.pulp champer and root canals.****************************

12-The test for testing the bur in which all the blades of the burs pass
through 1 point called:

a. Run out.***********************************
b. Concentricity.
c. Run out and concentricity.
d. None of above.

13-patient has a symphysis area and need maxillofacial surgery what is


the wire
a. 4 feet 8 gauge
b. 4 feet 28 gauge
c. 6 feet 20 gauge
d. 6 feet 26 gauge***********************************************

14-To check a perforation in the desk of the TMJ. we need: Cranial


imagery.
A.Arthrography."*************************
B.Traditional tomography.
C.Computerized tomography.

15-Oral herpes caused by which virus: ( corrected file )


a-herpes simplex type 1*********************
b- herpes simplex type 2

16-Condyle developed by :
A.Intramembranous ossification.
B.apposition and Endochondral ossification.*************

17-60 year old patient came to the clinic complaining of excessive


movement of denture. On examination there is elevation of anterior end
when u press o the distal end. TTT ( shaheeen )
A. Reline****************
B. Rebase
C. Remake
D. Denture adhesive

18-Question about Cemental dysplasia. ( shaheeen ) Clinical scenario.(


how to know ) the only case with : Female of African-American with a
radiolucency and the mandibular of the anterior teeth , teeth are vital )

19-Water irrigation device

A.prevent plaque formation


B.completely removes plaque
C.dilute bactetial toxins************

20-intracanal medication
a.Formacresol************************

21-Amount of daily wear of amalgam ingested in the body:


1 – 3 μgs/day of mercury.*********************
10 – 15 μgs /day of mercury.
25 μgs /day of mercury.

22-Patient comes to your clinic complaining that the denture become


tight, during examination you notice nothing, but when the patient stand
you notice that his legs are bowing (curved). What you suspect:
a.Paget’s disease.

23-Case scenario on ( Central giant-cell granuloma ) benign tumer affect


women and is more in the mandible , Micrograph of a central giant cell
granuloma showing the characteristic giant cells with surrounding cells
that have nuclei that are dissimilar to those in the giant cells.

24-picture of bluish swollen glazed lesion on tip of tongue and laterally


in 25 years’ female:

a- Sq. Cell
b- Hemangioma*************
c- Neuro fibroma
d- Lipoma

25-Needle used for aspiration cytology


19 gauge.

26-flouride which we use in clinic doesn’t cause fluorosis;


a-teeth already calcified

27-65 year old patient complains of pain sharp and lingering in Buccal
mucosa. It goes away and comes back spontaneously at night ,
stimulated with cold (when going outside ), pt is edentulous ,has had
extractions in the past. Pain is on one side
of face and does not cross the lip line:

A.trigeminal neuralgia **************


B.Bell's palsy
C.myofacial pain.

28-Porcelain shrinkage after firing:


A.1-5 %.
B.5-10 %.
C.10-20 %.***********

29-Instrument which use for grasping a tissue when remove thick epulis
figuratum:

A.Allis forceps.*************
B.Adson forcep.
C.Curved hemostat.
D.Stilli forceps

30-After finish class v glass ionomer cement we do finishing with:

a.Pumice slurry.
b.Aluminum-oxide disc.**********************

31-oval radiolucency between roots of upper central incisor

A.Incisive foramen**************
B.Radicular cyst
C.Granuloma
D.Absess

32-During mentoplasty/ genioplasty , doctor should take care for injury


of what nerve:

A.Lower branch of the facial nerve

33-After trauma a tooth becomes yellowish in color, this is due to:

A.Necrotic pulp.
B.Irreversible pulpitis.
C.Pulp is partially or completely obliterated.******************
D.Hemorrhage in the pulp.

34-Treatment of internal resorption involves:

A.Complete extirpation of the pulp to arrest the resorption


process.**********************
B.Enlarging the canal apical to the resorbed area for better access.
C.Utilizing a silver cone and sealer to fill the irregularities in the resorbed
area.
D.Filling the canal and defect with amalgam.
E.Sealing sodium hypochlorite in the canal to remove the inflammatory
tissue necrotic in the area of the resorption.

35-xray give real root length:


Parallel tech. *********
Bisecting tech.

36-The tip of size 20 endo file is:


0.02 mm.
0.2 mm.************

37-why calculus should be removed in perio diseases. ( shaheen )


A. To avoid plaque adherence *********************
B. Because its an etiological factor of periodontal disease
C.it irritates tissues
D.to maintain good oral hygiene

38-How will you asses oral hygiene / efficiency of oral hygiene methods
prescribed to the patient? ( shaheen)
a. Plaque index****
b. Calculus index

39-Premature contact between upper and lower ant. teeth in eccentric


occlusion while there is absolutely no contact on the centric occlusion.
So the management is by grinding of:
A.incisal edge of ant. max. teeth.
B.Incisal edge of ant. man. teeth.
C.Inclination of ant. max. teeth lingual.****************
D.Inclination of ant man teeth.

40-Patient presented to you after fitting the immediate denture 5 – 10


months, complaining pain and over tissue in the mandibular, what is the
diagnosis:
A.Epulis fissurment.**********
B.Hypertrophic frenum.

41-Biological width after crown lengthening


1 mm.
2mm.
3mm.***********
4mm.
42-Wax inlay which type contain in much gradient?

Paraffin wax.***********************************
Bee wax.

43-Pt. needs complete denture u take impression with irreversible


hydrocolloid & poured it after more than 15 min. the cast appears
smooth & chalky the reason is:

a. Dehydration of the impression. ***


b. Expansion of the impression.
c. Immerse the impression in a chemical solution.

44-what is the type of wax used to verify the occlusal reduc on for full
veneer
restoration

A. Onlay wax
B. Lowa wax
C. Utility wax*****************

45-Main use of dental floss:

a. Remove food debris. ( in files )****


b. Remove calculus.
c. Remove bacterial plaque. ( what I have chosen )****

46-Concentrating of acid used in etching porcelain veneer:


9.6 % hydrofluoric acid.*********
35 % phosphoric acid.
37 % phosphoric acid.
37 % hydrflouric acid.

47-Dental plaque is formed after:


a. 6 hours.*********************
b. 12 hours.
c. 24 hours.
d. 48 hours.

48-At the begining of the operation day in the clinic, you should start the
water/air spray for 2 minutes in order to get rid of which type of
microorganisms :
a. pseudomonas aurignossa *****************
b. Streptococcus mutans.
c. Streptococcus salivarius

49-High mylohyoid crest in patient for complete denture, the surgeon


must avoid vital structure which is/ during preproesthetic surgery of
mylohyoid ridge reduction:

Lingual nerve.*****************
Mylohyoid nerve.
Long buccal.
IAN
50-Drug used to decrease saliva during impression taking is:

a. Cholinergic.
b. Anticholinergic.************
c. Antidiabetic.
d. Anticorticosteroid

51-Dentigerous cyst treatment >>>>> Eneculation

52-Components of the forceps >>>>> a.Handle – hing – peaks .

53-Most common malignant salivary

A.Mucoepidermoid carcinoma****************
B.Plemorphic adenoma
c .adenoid cyctic carcinoma

54-Subgingival scaling and root planning is done by:


a. Gracey Curette.***************
b. Hoe.
c. Chisel.

55-which used for special area:


a. Gracey*******************
b. universal scalar

56-10-14 yrs. pt. excessive plaque and calculus, what is the best
ultrasonic to be used ( corrected file )

A. Piezoelectric**************
B. Magnetostrictive
C. Ultrasonics

57-A 21 years old patient who has iron


deficiency anemia, difficulty in swallowing,
with examination of barium sulphate, you
found:

A.Geographical tongue.
B.Burning mouth syndrome.
C.Plummer vinson syndrome.************
D.Diabetic patient.

58-Established gingivitis >> 14-21 days

59-Minimal distance between two implants


3 mm

59-Three years old pt. came to clinic with his parents he has
asymptomatic swelling bluish in color fluctant in midline of palatal
raphe, diagnosis is :

A.Bohn's nodules.
B.Gingival cyst.
C.Lymphepithelial cyst
D.Herps semplex virus.
E.Epstein's pearls.********************

60--5 years child with bilateral loss of deciduous molars & the anterior
teeth not erupted yet, the space maintainer for choice is:
A.lingual arch.
B.Removable partial denture.
C. Bilateral band and loop.( I have chosen this )*
D.Non removable partial denture

61-Pedo, has trauma in 11, half an hour ago, with slight pulpal exposure,
open apex, treatment is:

A.Pulpotomy.
B.Apexification.
C.Direct pulp capping .*************************
Extraction.

62-pt 5 y removed his upper central what to do:


A.No ttt *****************
B.Space maintainer
Crown
C.Maryland bridge

63-CONCENTRATION of sodium fluoride:


A.0.05ml daily*********
B.0.5ml twice a week
C.2 daily

64-Amount fluoride 5 year old, 0.5ppm water


fluoridation:
A.0.25***********************
B.0.5
C.1
D.1.25

65-stainless steel crown ready to cement but you find small open margin
from buccal what to do
A.Crimbing wz plier 112
B.Crimbing wz plier 114********************
C.New crown

66-Jeuvenile periodontitis treatment ? >> Tetracycline

67-What does dolicocephalic means:


A.Long skull***************
B.Short skull
C.Long face
D.Short face

68-fuctional appliance:
a.bionatar*********************************************************************
B.ant post bite block
69-what make priority to private clinics than community clinics
A.Need of assistance*************************
B.Insurance

70-FROM PPE: ( Corrected file )


A.uniform
b-mask*****

71-What’s the diagnosis of this picture ( Corrected file )


A.Dens evagenation
B dens invagention********************************************
C. Crown dileceration

72-pt. came to ur clinc have an painless ulcer on the lip , which begin
last 6 weeks as elevated border with deep center ulcer developed very
quickly during first 4 weeks then

73-( slowly growing or stop growing ) have no history of truma but the
pt. works outside under exposure of the sun. Biobsy reveals PMN &
acanthotic exudate, what is the diagnosis:
a. squamous cell carcinoma.
b. keratoacanthoma.*********************************
c. verrucus cell carcinoma.
d. mucoepidermoid carcinoma.

74-Optimal & minimum crown root ratio and minimal acceptable ratio (
Corrected file )

A.1:1 and 2:3 respectively.


B- irrelevant as long as there is no mobility.
C- 3:2 and 1:1 respectively.
D- 2:3 and 1:1 respectively. *************

75-pt with plasma cholinesterase deficiency , which anesthesia you


use?
A.procaine
B.prilocaine**************************************
C.procaine with vasoconstricter
D.another ester type

76-Case for impacted upper bi lateral canine ( corrected file )


A. Periapical
B. Dentigerous****************
C. Residual

76-Question ( I don’t know the answer )


Qst. amelogenesis imperfecta which type of ortho brackets do u
suggest:
a.invasilgn ( I have chosen this one )*
b.metalic
c.lingual bracket.

77-max denture (I don’t remember the question first time I have seen it ?
) >> extends 1-2mm beyond vibrating line

78-The matrix band should be above the adjacent tooth occlusal surface
by: ( corrected file )
a. 1 - 2mm. ***
b. 2 - 3mm.
C. 2.5 - 3.5mm.
D. Below to it

79-She told me that q talking about patient restored his tooth by


composite
restoration and came after one week with lighter color of composite.
Lighter color
caused by???

A- Insufficient light curing. ( I have chosen according to the file )*


B- improper isolation.
C- water resorption.

80_Loose enamel rods at the gingival floor of a class II amalgam cavity


should be removed using :
a. Straight chisel.
b. Hatchet.
c. Gingival curetla.
d. Gingival marginal trimmer.**************************

81_ The cement material with uniform filmthickness :


a. Zinc oxide
b. Resin
c. GI
d. Zinc phosphate********************

82-Pt. with complete denture complains from tightness of denture in


morning then becomes good this due to:

a. Relif of denture. ******************


b. Lack of cheeck elastisty.
c. Poor post dam.

83- Overcontouring of restoratn enhances


A esthetics
B cleansing action
C plaque accumulation**********************

84- After gingivectomy surface epithelisation occurs in


A.3 days
B.5-14************
C.14-21
D.Over a month

85- Proxy brush is used with which type of furcation ?


a- type 1
b- type 2
c- type 3******
d- type 4
Proxy brush is used with for furcation type 3 and embrasure type 2

****The rest I don’t remember

100-Last question in my exam ϋ


Seibert , 1983 classified , Class II is ?

a.buccolingual loss of tissue with normal apicocoronal ridge height

b.apicocoronal loss of tissue with normal buccolingual ridge


width******************

C.combination-type defects

EXAM – 21

1)patient suffered from swelling and pain at the meal time ?


a.Sialothisis **
b. Pain from Impacted 8
c. Pain from Fpd

2)During mentoplasty, doctor should take care for injury of what nerve:
a. Mental nerve.
b. Lingual nerve.
c. marginal mandibular nerve

3)minimum space between 2 implant


a.3mm**
b.4 mm
c.5mm
d.6mm

4) Gold standard for measuring malodor :


a. heliometer *
b. organoleptic
c.gas chromatography

5) Bacteria responsible for Caries initiation and progression:


streptococus mutans and lactobacilli**

6) Pic of compound odontoma

7) Treatment of Fucation type II involvement with non graft regenerative


technique:
a, osseous coagulum
b. GTR**

8)Rubberdam application:
4 contact points between clamp and tooth**

9)Discoid/cleiod instrument used for:


a. carving anatomy **
b. finishing
c. remove amalgam flushes
d.precarving…

10)Treatment ameloblastoma:
A.hemisection of mandibule with condyle
B.resection with free bone margin **
C.enuculation

11)long story about cyst with unerupted tooth ?


Dentigerous cyst**

12)PA x-ray for the upper ant maxilla (not clear) ?


a.supernumerary tooth*?
b.compound odontoma
c.complex odontoma

13) dye that is used with toludene blue to differentiate between cancer
and normal cells
a. methylene blue
b. congo red
c.lugol****

14) .acid which is more potent cariogenic is?


a.acetic acid
b. lactic acid**
c.teichoic acid

15) disinfectant of dental chair after HBV:


1. iodoform & hypochloride
2. formaldehyde
3. ethylene oxide gas
4. 400/ ethyl alcohol/detol
a- 1&2
b- 2&3
c- 3&4
d-1&2&3

16)tt of short margin crown? Remake**

17) stainless steel crown ready to cement but you find small open
margin from buccal what to do
A. Crimbing wz plier 112
B. Crimbing wz plier 114**
C. New crown

18) second preoperative x-ray in implant:


a.anter active computed tomography.
b. complex computed tomography.
c.MRI

19) The aim of treatment maintenance is:

20) Study of oral health on pregnant women


a-Cohort study
b-Cross sectional
c-Case control
d-Observational

21) Diagodent used for caries


A.deep*?
B.proximal
C.superficial

22)best time for the treatment of thumb sucking?


a.before the eruption of lower molar
b.encourge the parents to stop the habit
c.before the eruption of upper anteriors*?
d.reasurre ….

23)pic of mandible where the needle touchs the condyle,name of the


technique?
a-Gow gates technique**

24)case about condinsing ostitis

25) Since cavity deisn for composite should be conservative as


much as possible ; concept is acceptable?
a.Conventional amalgam**?
b.Bevel amalgam
c.Extension for prevention
d.Conservative with special preparation

26)pt with renal transplantation has lesion on palate (white dots ) Pic>>>
a.pseudomembranous canidiasis***
b.erythmatous candidiasis

27)effect of water or alcohol on zinc oxide eugenol impression material


a.slow
b.delay
c.stop
d.speed

28)to increase working time of alginate?


a.cold*
b.hot

29) Composition of gutta percha?


70% zinc oxid, 20% gutta percha

30)function of rest in RPD?


Support**

31)The most destructive occlusal interference is


A. Centric
B. Working
C. Non working**
D protrusive

32)Pt Came with fracture ,surgeon decided to do fixation,what is the size


of the wire that should be used?? A:4 inch 22gauge
. B:4inch 20 gauge.
C:6 inch 26 gauge**

33) patient who has un-modified class II kennedy classification, with


good periodontal condition and no carious lesion, the best clasp to use
on the other side:
a. Reciprocal clasp\aker's clasp.
b. Ring clasp.
c. Embrasure clasp.**
d. gingivally approaching clasp.

34) class 2 Kennedy good oral hygiene and free of caries:


A.Circumferential clasp
B.Ring clasp
C.Back action clasp

35) Smokers :
Oral cancer
Control
Smokers
60%
40%
Non smokers
10%
90%
What is the percentage?
13.5**

36)case about nasopalatine cyst

37) Dental materials are classified as


A. ceramic, polymers and composite
B. ceramic, polymers and alginate.
C. ceramic, polymers. Cement
D. ceramics, metals, polymers,composites**

38)case about reverible pulpitis

39) siebert ridge classification for apicocoronal loss>>>> class II**

40)role of varnish?
a. prevent discoloration
b.prevent leakage
c. insulator

40)-anasthesia? lipid solubality of unionized ions**

41) Decalcification of enamel in ortho ??


A: around the bracket.**
B: under the bracket

42) -c shaped canal found in >> mand 2nd molar

43) post extraction pain?


a. resolve after 5 day
b.treat with narcotic analgesics**?
c.sorry I forgot the other options

44) pt with high mastication and needs aesthetic posterior restoration,


whats the best choice
a- Zinc polycarboxilate
b- GIC
c- Composite with bevel
d- Composite without bevel**

45) A tooth with fracture cusp dentine involves what is the status of pulp
in this case?
a. hypersensitivity
b. reversible *?
c. irreversible

46) split dam technique used with


Single crowned tooth
Posterior molar
Fixed partial denture field

47) nursing caries in child :


A. Lower incisor
B. Upper incisor**
C. Lower molar

48)Pt missing 4 ant teeth and he want to replace it by Fpd,


What is the abutments for this case?
A-Rt & Lt canine
B-Rt & left canine and premolars**
c-both canines and left premolar
d-both canines and right premolar

49)t o plane the facial and the lingual wall of enamel , which enamel will
use :
A. Enamel hatchet**
B. Gingival trimmer
C. Chesil
D. Plane line angle

50)picture of 8 years old pt with fractured tooth , no radiographic signs


and not tender to percussion ,what is the treatment?
a.pulpectomy
b.partial pulpetomy with calcium hydroxide
c.pulpetomy with calcium hydroxide
d.calcium hydroxide and restoration

51) A root seldom has 2 pulp canals


A. mesiobuccal root of upper molar
B. mesiobuccal of lower molar
C. distobuccal root of upper molar**

52) Xray to see relation between impacted tooth (sorry I forgot the exact
q):
1) PA*?
2) OPG
3) occlusal
4) lateral ceph

53) Distance between cephalogram and patient


A. 5 feet**
B. 6 feet

54) many q's about preventive resin restoration

55) What’s the name of the caries system?? 1: MTD 2:MDF 3:DMF **

EXAM – 22

1.bacteria for initiation and progression of caries…. Streptococcus and


bacillus

2.Avulsed tooth 30 min …soak in Naocl then NaF

3.Factor/agent responsible for initiation of caries in oral cavity..


a.strepto b. bad oral hygiene **

4.ulcer in lower labial sulcus after few days of new denture insertion ..
A,under extended flanges
b.overcontoured flange
c.overextended flange
d.high occlusal force

5.child came with his mother playing with instruments and becoming
restless and then aggressive how can we manage him hyperactivity
disorder : a. progressive management b. talking to him in language for
children c. describe the steps for treatment step by step

6.sensory nerve supply to upper lip: a. facial I forgot other choices!

7. After upper molar injection a colorless extraoral swelling .. inside


which artery the inject was? Zogomatic..maxillary..facial..

8. submandibular gland calculs causing pain during meal which x ray


used to detect :
Occlusal..panoramic..and other choices forgot!

9.6 years old child with Bluish swelling above lower 6 whats the proper
managemen:
a. incision and extraction
b.incision and induce eruption
c.observation and small incision to expose the molar

10.which LA agent for pregnant lady in her 2nd trimester … lidocaine


11.spedding principle: Stainless steel crown selection

12.Gun shot to mandible fixation technique a.christian b. keans

13.nefidipine induced gingival enlargement why we do gingivectomy ?


..elimination of pseodopocket

14. which type of dentine shiny and upper layer is calcified –sclerotic-
secondary-tertiary-reparative >> I don’t remember the words in question
but that was the idea!

15.Cause of pain in irreversible pulpitis.. a. increase intrapulpal pressure


b. degeneration of odontoblasts and other option I forgot but I think it
was the answer!!

16. a question to describe the sensation towards a stimuli to pain


receptors, in the patients own feeling I think its just Pain.

17.LA mechanism
a.blocking impulses
b.increase pain threshold
c.decrease pain threshold

18.Test to differentiate between periapical abcess and periodontal


abcess
..radiograph..vitality..

19.condition that does reach defined goals reflect effectiveness !! ethics


question !!
a. controlled condition
b.uncontrolled
c.normal

20.Molar abutment for a looong span fixed denture patient feels pain and
discomfort :
a.break in connector?
b. vertical tooth # **?

21.bone tempreture during implant cuse bone damage …46 C for 1 min*

22.mercury ingested daily from amalgam …1-3ug

23. % of copper amalgam which reduce gamma 2 phase ..13

24.teeth in alveolar cleft deviate to.. non cleft side

25.impression disinfectant : %glutaraldehyde (but check the


concentration of it )

26.truma to child with avulsed tooth put im milk since 30 min how long
we splint ?
By days : 1-7 days … 7-15 days …21 days

27.Aliminum foil test for which strilisation …ultrasonic?


28. newly postgraduate dentist took an impression to screening room
how should he disinfect the impression : a. surface only b.surface and
tray c. surface and bottom and tray

30.something regarding stage of porcelain I am sry don’t remember but


I think answer was dough stage !!

31.RME (maxillary expansion) for child after few days mother called
clinic that a diastema appears :
a.continue expansion normally ****true
b.reverse the screw
c.ask orthodontist
32.perio weak tooth ortho movement… light
33.decalcification in ortho .. a.around brackets b. under brackets

34. least dimensional stable.. a.metal b.plastic c.ceramic..(I don’t


remember the Question !)

35.pit and fissure Alternative restoration


a.GIC
b.compomer
c.composite

36. system depend in continuous waves:


a.Obtura1
b.obtura2
c.ultrafil
d. system b

37. file #20 tip ? 0.2

38.occlusion in which posterior teeth are in occlusion and anterior also


(if possible) balanced occlusion.

39.material used in oral cavity binds to all surfaces and has ability to
release ions with time and has decrease microbial affect
a.Gic b.flouride

40.prophylaxis antibiotics when given .. prosthetic valve

43. pt congestive heart failure management :a. position chair in upright


..b. treat as normal pt .. avoid O2

44.unresorbable suture material : silk

45.subgingival instrument … gracey

46. Xray about cleidocranial dysplasia

47.case picture has upper complete denture with porcelain lower class
one kenedy with the anterior natural has wear defect what is called:
a.attrition
b.erosion.
c abrasion
48. how to make LA injection less painful :
a.topical
b.strach tissue
c.slow injection
d.gauge 25
1.a b c ***
2.a c d
3.c b d
4.a b d

49. after bracket removal discoloration whats the management ..full


crown

50. central incisor with full veneer crown after sometime came need
endo for same teeth ..lingual access cavity made through the veneer
crown whatis the proper way to restore the cavity? Remake another
crown??

51.Xray with impacted lower premolar with a cyst what is the diagnosis..
dentigerous or eruption cyst?

52.after having removable ortho appliance difficulty in which which


voul?...Lingual for several weeks?

53.condyle growth and formation … apposition and endocondral

54.Time for gingivitis.. 14-21 days

55.in pedo which nerve should anesthetise for upper molar.. superior
posterior AN ??

56.Ghost teeth : dentinal dysplasia

57. Important in rotary endo.. precoronal enlargement

58. Most common malignancy in oral cavity… SCC

59.which are opposing retentive are in RPD ..reciprocal?

60.For good oral hygiene which clasp? Circumfrential

61.instrument has positive rak angle : without saying more


a.kfile
b.reamer
c.?? but h file was not there

62.Malar bone which case?

63.Antiboitic inhibits cell wall ..penecillin

64. first formation of tooth bud ..6 weeks embry

66.most important property in fissure sealant material ..vicosity


67.decay index .. DMF

68.percentage or stannous fluoride

69. what is the thist occlusal refrence point ..a .nason b.condyle. c
occlusal plane
.
70.whicle retreatment of endo after GP removal dentist tries to insert file
to working length but lost it?? ..ledge

71.overhanging restoration affects.. a. contact area b. periodontal


health**

72. apicectomy cut angle : a.perpendicular b. acute c.obtuse

73. indication of maintaining good oral hygien by ..: decrease plaque


index.?

74.why denture put in solution after remval in bed time : to avoid fracture

75.Most common tooth has abcess a.molar b. premolar c.incisor d.


canine

76.why caries in adult more than pedo

77.Allergy from composite because of with content : a . monomer

78.facial fracture causing list of signs I forgot but it was zygomatic


complex fracture same file

79.Most common irrigation:Naocl

80.Lap pontic disadvantage.. Tissue irritation

81.measure attachment loss :


a.from free gingiva to pocket
b.from cej to pocket.
from cej to mucogingival junction

82.patient came after 48 hrs of complex crown fracture of incisor


exposed pulp management:
a.rct
b.partial pulpotomy
c. complete pulpotomy
d.Dpc

83.pt came to clin had checkenpox dr said it affected calcifying and


eruption .which stage of development affected :
a.proliferation
b.histodiffrentiation
EXAM – 23

1. Blood supply to TMJ?


a. External carotid artery

2. Anesthesia for lower posterior teeth bone and tissue


a. Gow gates ***
b. Akinose
c. Inferioor alveolar nerve block

3. Diameter of arteriole of dental pulp in micrometer


a. 35-45??
b. 15-25
c. 5-10
d. 0.3-1

4. Bacteria not found in pericoronitis


a. bacteriodes
b. peptostreptococcus
c. haemopis??
d. Prevotella

5. Management of root that got displaced into the maxillary sinus


a. Leave and observation
b. Administer antibiotic
c. Remove it as soon as possible ***

6. Pt with buccal space infection related to lower molar what is the emergency
management
a. Extract the tooth and antibiotics later
b. antibiotics
c. Incision and drainage
d. Refer to maxillofacial surgeon??

7. How many carpules can you give from lidocaine 2% with 1:100 000
epinephrine to a child who weighs 80 kg in mg
a. 560***
b. 450

8. Uncommon pattern of mandibular permanent teeth eruption


a. Lower central before 1st molar
b. 1st molar before central
c. Lower 7 before 2nd premolar ***

9. Ideal medium for avulsed permanent


a. HBSS

10. Best media for avulsed tooth in child


a. Cold milk ***
b. Saliva
c. water

11. 5 yrs old lost his upper central what to do?


a. Nothing ***
b. Polycarboxylate crown
c. Space maintainer
d. Resin bonded bridge

12. Maxillofacial surgeon wants to put 2mm plate what is the size of the drill he
will use?
a. 1mm
b. 1.5mm ***
c. 2mm

13. Amalgam cupper concentration to eliminate gamma 2 phase?


a. 13%

14. Retention in amalgam filling


a. Walls convergence ***
b. divergence

15. Emergency after giving LA what is the most common reason?


a. Epinephrine
b. Stress??

16. Waldeyer's ring


a. Lymphoid tissue ***
b. neural

17. Pt with gun shot wound condyle is injured and surgeon want to replace it
with graft from?
a. Anterior iliac crest
b. Post iliac crest
c. Costochondral??

18. True about dentinogenesis ?


a. Dental lamina becomes clear
b. Dentinal matrix is involved in the process ??

19. Pt with dog bite in the face he is stabilized and tetanus shot given what will
the maxillofacial do?
a. Irrigate with Hydrogen peroxide
b. Lacerate the wound so it would heal primary healing
c. Place iodine inside
d. Suture in layers ??

20. Autoclave?
a. dry
b. Wet steam sterilization ***

21. Perio furcation involvement II with recession what is the regenerative


procedure without graft
a. GTR ***
b. Osseous coagulum

22. Floss
a. Removes plaque
23. Pt with anterior cleft lip and palate at what time did the problem develop?
a. 8-11 intrauterine life ***
b. 4-6

24. File with positive rake angle


a. K file ***
b. Profile
c. Protaper
d. K reamer

25. Patient swallowed fluoride tooth paste what the best first response
a. Drink milk or similar high calcium product ??
b. Take to emergency

26. Material in caries dye


a. Propalyne glycol

27. What is a suitable combination of files


a. Yellow-10
b. Red-25 ***

28. Pt had RCT and 1 week after obturation he had persistent pain, in x-ray filling
is short what to do?
a. Give analgesic
b. Start retreament
c. Give antibiotic and analgesic
d. Wait and observe

29. Fluoride in special need pt


a. APF gel
b. Stannous fluoride solution
c. Sodium fluoride solution
d. Fluoride varnish ***

30. File length


a. 21,25,31

31. Access cavity in upper central and lateral incisors


a. Oval ***
b. triangular

32. concentration of fluoride in water


a. 1 ppm

33. 7 year old with unilateral functional crossbite on right side due to mandible
shift when opening? What to do
a. Unilateral maxillary expansion on right side
b. Unilateral maxillary expansion on left side
c. Wait and observe??
d. bilateral maxillary expansion

34. normal or excessive occlusal forces are placed on teeth with compromised
periodontal attachment
a. primary occlusal trauma
b. secondary occlusal trauma ***
35. Pt 67 years old with bony lesion in the mandible lower face is enlarged lower
lip is protruded , hypercementosis, thickened mandibular cortex, in x-ray mixed
RO and Radiolucent lesion in mandible area
a. Paget’s disease ***
b. Fibrous dysplasia
c. Florid osseous dysplasia

36. Pt 22 yrs old with bluish dots in the palate, rash for the previous 4 days,
thrombocytes count 25 000?
a. Leukemia
b. Thrombocytopenia purpura ***

37. X-ray case of dentigerous cyst

38. Step before final impression


a. Special tray fabrication

39. Cause of shrink spot in crown casting?


a. High temperature
b. Wide sprue
c. Long sprue handle??

40. Pt with lost lower 2 premolars history reveals that he has xerostomia how to
replace his teeth?
a. RDP??
b. Implant fixed bridge ??
c. Convetional bridge
d. Resin bonded bridge

41. Patient with ulceration all over his gingiva especially in the interdental papilla
area, halitosis
a. Necrotizing ulcerative gingivitis?

42. Gingival margin trimmer is used for


a. Beveling gingival margin of class II

43. Patient came for a denture very anxious …


a. Hysterical pt

44. Denture missing a component that prevents tissue wards movement which
is causing problem the pt what is the missing part?
a. Occlusal rest

45. Most common salivary gland tumor?


a. Pleomorphic adenoma

46. Most common benign salivary gland tumor


a. Pleomorphic adenoma

47. Space mesial to upper canine in primary teeth


a. Primate spaces

48. The most important factor in oral habits causing maloclussion ?


a. frequency
b. duration??
c. force
d. direction

49. peasoreamer is most useful in


a. removing vital tissue from thin canal
b. removing cotton from canal??
c. Preparing coronal part of the canal

50. Impression material with the highest stiffness


a. Polyether

51. The duration of anesthesia is affected by:


a. removal time
b. bond strength between the anesthesia & the nerve
c. anesthesia bond strength & removal time??
d. The duration of administration

52. Difference between Gracey and universal curette:


a. Gracey used for cutting in specific area while universal is in any area.
b. Gracey has one cutting edge while universal has two.
c. gracey has 70 offset, universal 85
d. Section of gracey is hemicircular and in universal triangular.
1. A,b,c??
2. A,b
3. A,b,d

53. Cerebrospinal fluid can be recognized because it contains?


a. high protein
b. high glucose??
c. Beta 2 transfirrin
d. glucose oxidase

54. Rubberdam application:


a. 4 contact points between clamp and tooth Age of the patient

55. NiTi feature


a. Shape memory

56. Maxillary sinus opens in


a. Middle meatus

57. Contraindication for veneer


a. bruxism

58. Complete denture pt tissue over mandibular ridge is easily retractable how
to manage?
a. Minor surgical procedure to correct??
b. Impression will replace the tissue

59. Patient with high caries risk which smooth surface of permanent teeth is
prone to decay first
a. Facial of upper posterior
b. Lingual of upper posterior ??
c. Facial of upper anteriors
d. Lingual of upper anteriors

60. Pt with high caries risk, Stained questionable pits and fissures, what step do
you do for diagnosis?
a. Follow-up after 6 months
b. Exploratory opening of the pits and fissures ***
c. Composite resin filling
d. Seal the pits and fissures

61. For composite restoration conservative design


a. Amalgam preperation
b. Beveled amalgam preperation
c. Preparation following decay ***

62. Pt with discolored proximal surface, no cavity, good oral hygiene,


management?
a. Apply sealant
b. Restore
c. Fluoride ***

63. Pt with decayed #36. Teeth #16,#26,#46 are stained but sound what is the
management?
a. Apply sealant

64. Endo diagnosis case?

65. Patient with two moderately deep carious lesions, pulp response is normal
what to do?
a. Place permanent restoration in one visit??
b. Place permanent restoration in two visits
c. Place temporary and then permanent in 1 week

66. Definition of reversible pulpitis

67. Pt has prolonged pain on cold, sometimes spontaneous pain diagnosis?


Irreversible pulpitis

68. Concentration of NAOCL? 5.25%

69. AH plus different than AH 26? no formaldehyde release

70. Indication for doing indirect pulp capping in pedo? Caries penetrating
dentine

71. Brushing technique that uses side of bristles of the brush? Modified
stillman?

72. Pt with calculus advised to use brush frequently why? To break plaque
formation ..

73. Base of carious lesion is always toward


a. Enamel
b. DEJ ***

74. Treatment of acute pericoronitis?


a. Antibiotic
b. Excision and antibiotic
c. Curettage and irrigation

75. Pt came with pain prolong pain on cold on a specific molar, you tried to do
cold, hot, electric pulp test but couldn’t reproduce the symptoms what should
you do for pulpal diagnosis?
a. Percussion test
b. Anesthetic test
c. Drilling test ***

76. Gutta percha is mostly composed of


a. Zinc oxide

77. Pt lost his lower 6 and you want to replace it with a bridge, upper 6 is
extruded. Where will interference most probably take place?
a. Protrusive movement ??
b. Working side interference
c. Non-working side interference

78. The vasoconstrictor affects LA in terms of?


a. Increase duration and intensity?

EXAM – 24

2.When is Thermo plasticized GP used


a.Canal with irregularities***
b.Curved canals
c.Where lateral condensation is difficult
d.Canals with open apex

3. What are the disadvantages of mcspadden technique in obturation:


a. Increase time
b. Increase steps.
c. Difficult in curved canals***

4. Retention of Complete denture – From Posterior Palatal Seal***

5. Posterior palatal seal is ?


a.Distance between hamular notch
b.junction of hard and soft palate***

6. 10y ingested 10mg fl. What is the immediate management


a. Drink milk withe calcium***
b. Go emergency
c. another option that is not related
(there was no option of do nothing otherwise would have chosen it)

7. Scaling in epileptic motion?


a.ultrasonic
b.Piezoelecrtic
c.Magnetostrictive***
d.scaler

8. APF Conc
a.1.23***
b.12.3
c.123
d.0.123

9. - in upper 8 Impaction to avoid tear of gingival flap


a. Adequate size of flap
b. Flap include greater palatine nerve
c. Strong retraction of flap margin

10. 4th no. In instrument classification—> angle of blade***

11. A picture showing occlusal wear of lower teeth opposing PFM


crowned upper anterior teeth what is the diagnosis:
a.abrasion
b.abfraction
c.erosion
d.attrition

12. Percentage to prevent phase 2 amalgam copper = 13%

13. newly erupted premolars and molar>>>


a.fissure sealants***
b.preventive
c.fluoride
13. Question on characteristics of pagets disease
14. Question on cancer related to gardners syndrome in which organ
a.colon***
b.lung
c.heart
d.pancreas
15. epoxy brush = embrassure type II***
16. Most difficut tooth to anasthesize = lower molars***
17. c shaped canal found in = mand 2nd molar***
18.Mechanism of GTR technique
19.Chlorohexidne conc. = 0.12
20.patient with swelling & diagnosis is keratocyst, to confirm the
diagnosis they decide to do fine needle aspiration what needle gauge
used
a. 19
b. 21
c. 25***
d. 30
21. Tooth with external resorption
a.Necrotic***
b.irreversible pulpitis
c.reversible pulpitis
22. Pink tooth
a.internal resorption***
b.external resorption
23. The worst type of bone
a. Type I
b. Type II
c. Type III
d. Type IV***
24. exposed roots, caries free teeth but reversible pain with cold water =
Dentine hypersensitivity***
25. opg xray showing supernumerary teeth
a.ectodermal dysplasia
b.cleidocranial dysplasiac***
c. craniofacial dystosis
26. order of placing of winged clamp type rubber dam
a. clamp before dam
b. clamp aftr dam
c. clamp and dam together ***
27. Niti feature = shape memory***
28. Relative contraindication of NO2 = suppression of gag reflex (made
most sense out of all options that I cant really remember).
27. pic of palatal infiltration in anterior area anesthetizing which nerve
a.nasopalatine***
b.greater palatine
28. mercury ingested daily from amalgam = 1-3ug ***
29. how to make LA injection less painful :
a. topical
b. stretch tissue
c. slow injection
d. gauge 25
1.a b c ***
2.a c d
3.c b d
4.a b d
30. Time for gingivitis.. 14-21 days
31. Formation of cleft palates in which phase
a.6 weeks
b.8-10***
c.15

32. decay index = DMF


33. Ridge Lap pontic disadvantage = Tissue irritation***
34. fiber optic light can be used to localize the canal orifices by directing
the light to the pulp chamber, the orifices will be seen:
a. light
b. dark ***
c. yellow
d. white

35. Water irrigation devices is used to:


a. prevent plaque formation
b. removes plaque ***
c. dilute bactetial toxins
36. In hairy tongue which papillae increase in number
Filliform***
Fungiform
Circumvallate
37. Flouride should be applied in
2mins
4min***
6 min

EXAM – 25

1. best xray for proximal teeth ?

2.7 years old pt came with fracture in subcondylar area.


the other side have class 1 malocclusion , fracture side have class 2
malocclusion . manual reposition it shows class 1 . whats you tt? A.
open reduction . b closed reduction. c . follow up .

3. instrument with number 15 . 83. 8. 14 . which is for length of balde ?

4 . 3 years under general anasthesia , xray show small caries proximal


area D and E , tt ?
A. composite
B. amalgam
C.stainless crown
D. pulpotomy with stainless

5.convergence of walls amalgam

6 . pt 17 years started forming open bite , no sucking finger habit .


A. cleidocranial dysplasia
B. eagle syndrom
C. treacher collins
D .plummer vinson

7. pt had radiotherapy 4 years back . edentulous, need CD .


which material to use for impression
A. compound
B. plaster paris
C.zoe

8. standard and conventional gp difference

9. space between 2 implant

10. non resorbable suture , most used in oral surgery .

11. pic of fibroma

12. GP : 70 % ZOE 30% gp

13. why use low speed bur for temporary teeth ?

14. dentist romoves all caries in a tooth , than saw a small red point
bleeding lighly .
A . direct pulp capping
B. indirect
C. pulpotomy

15. Percentage of copper in amalgam which reduces gamma 2 phase?

16. crucial need for appliance to stop thumb socking


A. temporary dentition
B. early mixed dentition
C. late mixed
D. permanent dentition

17.
diabetic patient suffering 15 days came with swelling , erythema , pain
in the area of right mandibular molars ( no mobility) . xray ; moth eaten
appearance . diagnosis ?
which osteomyelitis?

18. oxygen flow rate per minute .

19. 7 years old , intruded max ant teeth after fall . tt

20. Pt have calculus and u want remove using device have elleptical
motion .. Which is it...?
A.ultrasonic
B.piezo
C.hand instrument
D.magnetic

21. Brush method with side parts of bristle activated ?

22. During 3/4th crown preparation on pm bur used to add retentive


grooves

23. character of irreversible pulpitis ?

24. Most potent vasodilator?


a.cocaine
b.procaine
c.tetracaine
d.atricaine

25. The substance in local anaesthesia cartridge responsible for prevent


oxidation of vasoconstrictor: A.sodium chloride solution B.sodium
metasalphate C.sodium salphate

26. bionator

27.When you give sedative inhalation for patient to prevent hypoxia u


give :
A-95% oxygen and 5% nitrouse oxide
B-90% oxygen and 10%nitrous oxide
C-85%oxygen and 15%nitrous oxide
D-100% oxygen and zero nitrous oxide
28.patient came to hospital with gun shot ,,the surgeon will make fixation
by : a- christian's technique b- keen's technique

29. Pt. got gun shot , question about graft used for condyle ?

30. best xray for parotid

31. swelling when eating . sialothitis

32. second best xray for planning and fixing implant

33. 2 layers of varnish under amalgam .

34.class 1 malocclusion , the vertical dimension


A. 1 mm less in rest position
B. 5 mm
C. 6mm

34. which part of periodontal tissue regenerate the last ?

** THE END **

You might also like